You are on page 1of 48

LOCAL GOVERNMENT (c) Quasi-public corporations are a species of private corporation but the

qualifying factor is the type of service the former renders to the public:
if it performs a public service, then it becomes a quasi-public corp.
I. General Principles
i) Examples: utility, railroad, warehouse, transportation companies.
A. Public Corporations
1. Boy Scouts of the Philippines v COA
(2) Funa v MECO
a) BSP is a public corp subject to audit. It is considered as an institution for
public interest or purpose created by law. (Art 44, NCC) It was created to (a) COA has authority to examine, audit and settle accounts of the ff:
serve a purpose in pursuit of a constitutional mandate. i) Government, any of its subdivisions, agencies and
b) Not all corporations which are not GOCCs are ipso facto private corporations; instrumentalities;
they may be public corporations. ii) GOCCs with original charters
(1) Public corps are treated by law as agencies or instrumentalities of the iii) GOCCs without original charters
government which are not subject to the tests of ownership or control and iv) Constitutional bodies, commissions and offices that have been
economic viability but to different criteria relating to their public purposes/ granted fiscal autonomy under the Constitution
interests or constitutional policies and their administrative relationship to
the government or any of its department v) Non-governmental entities receiving subsidy or equity, directly or
indirectly from the government
(2) It is under the Department of Education.
(b) Government Instrumentalities are agencies of the national government
c) BSP is a public corporation or a government agency/instrumentality with that, by reason of some special function or jurisdiction they
juridical personality. perform, are allotted operational autonomy and are not integrated
d) COA has jurisdiction over government instrumentalities within the department framework. E.g. Regulatory agencies, chartered
B. Municipal Corporations institutions, Government corporate entities or government
instrumentalities with corporate powers, GOCCs
1. Concept
(c) MECO is a sui generis corporation
a) Distinguished from GOCCs
i) MECO is a private non-stock corporation, entrusted by the
(1) PSPCA v COA government with the pursuit of unofficial relations with the people of
(a) PSPCA is not subject to control or supervision by any agency of the a foreign land whose government the Philippines is bound not to
State, unlike GOCCs. recognize. It is not a governmental entity
(b) Just because a juridical entity is impressed with public interest does
not mean, by that alone, that it is a public corporation. This class of (3) Carandang v Desierto
corporations may be considered quasi-public corporations, which
are private corporations that render public service, supply public wants (a) Ombudsman has jurisdiction over admin cases involving grave
etc. misconduct committed by the officials and employees of GOCCs and
the SB has jurisdiction to try and decide crim actions involving RA
3019 committed by public officials and employees, including
presidents, directors and managers of GOCCs
Page 1 of 48
(b) A GOCC is a stock or non-stock corp whether performing • Government of the Republic of the Philippines refers to the corporate governmental entity
governmental or proprietary functions, which is directly chartered by through which the functions of government are exercised throughout the Philippines,
special law or organized under the corp code is owned or controlled by including, save as the contrary appears from the context, the various arms through which
the government directly or indirectly, to the extent of at least a majority political authority is made effective in the Philippines, whether pertaining to the
of its outstanding capital stock or of its outstanding voting capital stock autonomous regions, the provincial, city, municipal or barangay subdivisions or other forms
i) Government must own at least 50%+1 stock for it to be a GOCC of local government.

(4) Republic v Paranaque • (2) National Government refers to the entire machinery of the central government, as
distinguished from the different forms of local governments.
(a) Instrumentalities are agencies of National Government not integrated
into the department framework, vested with special functions. They • (3) Local Government refers to the political subdivisions established by or in accordance
exercise autonomy in their operation but still part of the National with the Constitution.
Government. It is not a stock or non-stock corporation • (4) Agency of the Government refers to any of the various units of the Government,
(b) Clearly, the test of economic viability does not apply to government including a department, bureau, office, instrumentality, or government-owned or controlled
entities vested with corporate powers and performing essential public corporation, or a local government or a distinct unit therein.
services. The State is obligated to render essential public services • (7) Department refers to an executive department created by law. For purposes of Book
regardless of the economic viability of providing such service. The non- IV, this shall include any instrumentality, as herein defined, having or assigned the rank of
economic viability of rendering such essential public service does not a department, regardless of its name or designation.
excuse the State from withholding such essential services from the
• (10) Instrumentality refers to any agency of the National Government, not integrated
public. within the department framework vested with special functions or jurisdiction by law,
(c) It is not a GOCC because GOCCs need to be a stock/non-stock endowed with some if not all corporate powers, administering special funds, and enjoying
corporation. To be a stock corporation there must be shares divided operational autonomy, usually through a charter. This term includes regulatory agencies,
and are authorized to distribute the shares. chartered institutions and government-owned or controlled corporations.
(d) PRA is not a GOCC because although it has divided stocks, it cannot • (12) Chartered institution refers to any agency organized or operating under a special
distribute it. charter, and vested by law with functions relating to specific constitutional policies or
(e) GOCCs must meet the economic viability test—organized as stock objectives. This term includes the state universities and colleges and the monetary
corporation usually just like private crp authority of the State.

(f) Instrumentalities need not meet economic viability test. • (13) Government-owned or controlled corporation refers to any agency organized as a
stock or non-stock corporation, vested with functions relating to public needs whether
(g) Local Government cannot tax national government; instrumentalities governmental or proprietary in nature, and owned by the Government directly or through
are part of the national government. its instrumentalities either wholly, or, where applicable as in the case of stock corporations,
to the extent of at least fifty-one (51) per cent of its capital stock: Provided, That
government-owned or controlled corporations may be further categorized by the
• The Congress shall not, except by general law, provide for the formation, organization, or Department of the Budget, the Civil Service Commission, and the Commission on Audit for
regulation of private corporations. Government-owned or controlled corporations may be
purposes of the exercise and discharge of their respective powers, functions and
created or established by special charters in the interest of the common good and subject
responsibilities with respect to such corporations.
to the test of economic viability. [Sec 16, Article XII, 1987 Constitution]

Page 2 of 48
2. Classifications i) Tan v COMELEC
a) Quasi-corporations (1) Plebiscite must be participated in by the units affected. The
b) Municipal Corporations population of the mother province and/or the surrounding areas
of the proposed Negros del Norte must be included in the
(1) Elements plebiscite and not just those who are within the metes and
(2) Nature and Functions bounds of the proposed unit.
(a) Vilas v City of Manila (a) The surrounding areas are also affected because they lose
land population and a reduced income from the national
i) Corporate identity of Manila was not altered by the fact of the
government.
change in government.
ii) Navarro v Executive Secretary
ii) The legal entity of the City of Manila survived both its military
occupation by, and its cession to, the United States, and, as in law, (1) With respect to municipalities, cities and provinces, the three
the present city, as the successor of the former city, is entitled to indicators of viability and projected capacity i.e. income,
the property rights of its predecessor, it is also subject to its population, and land area are provided for.
liabilities. (2) When the local government unit consists of 1 or more islands, it
is exempt from the land area requirement if the LGU to be
created is a municipality or a component city—should also
• Every local government unit created or recognized under this Code is a body politic and apply to provinces because there would be a greater likelihood
corporate endowed with powers to be exercised by it in conformity with law. As such, it
that islands or group of islands would form part of a newly
shall exercise powers as a political subdivision of the national government and as a
created province.
corporate entity representing the inhabitants of its territory.
iii) League of Cities v COMELEC
(1) The share of local government units in the national taxes is not
(3) Requisites for creation, conversion, division merger or dissolution
a specific amount. It is determined on the basis of the
(a) Provinces population, land area and equal sharing. This is also
(b) Cities determined on the number of existing cities.
(c) Municipalities iv) Miranda v Aguirre

(d) Barangays (1) A plebiscite is required for a conversion from independent


component city to a component city.
(e) Autonomous Regions
(a) there would be a material change in political and economic
i) ARMM rights of the LGU and the people therein. Thus consent of
ii) CAR the people directly affected is required.
iii) Administrative Regions (2) There would exist a substantial change e.g. city mayor will now
be under the supervision of the provincial governor.
(f) MMDA
(3) Constitutional requirments for conversion
(g) Cases

Page 3 of 48
(a) Income, population, and land area Mindanao and (2) which provinces and cities, among those
(b) Plebiscite by units affected. (no better check than the enumerated in R.A. No. 6734, shall compromise it.
people themselves) viii) Disomangcop v Datumanong
v) Umali v COMELEC (1) Organic acts are more than ordinary statutes because they
(1) “Directly affected” includes those LGUs whose boundaries are enjoy affirmation by plebiscite. Hence, they cannot be amended
to be altered and economically affected. by an ordinary statute; they must be submitted to a plebiscite.

(a) Conversion into HUC would entail: reduction of revenue (2) Idea behind autonomous regions is to allow the separate
allotment of the province; and the new unit would not be development of peoples with distinctive cultures and traditions.
under provincial supervision but supervision direct from the These cultures must be allowed to flourish as a matter of right.
president (3) Creation of autonomous regions does not signify the
(2) Legislative power to declare a component city as HUC by the establishment of a sovereignty distinct from that of the
President was delegated through the LGC. May be done upon Republic.
proper application and once the minimum requirements are (4) Regional autonomy is the degree of self-determination
satisfied exercised by the LGU vis-a-vis the central government.
vi) Bagabuyo v COMELEC (5) Autonomy requires decentralization—national government
(1) Constitution and LGC expressly require a plebiscite to carry out delegates power to subnational units to widen its reach
any creation, division, etc of an LGU; but there is no need, as a (a) Deconcentration: administrative; devolution; political,
general rule, for a plebiscite for legislative reapportionment. abdication of power
(2) Legistlative district is not a political subdivision which carries (6) The devolution of the powers of the DPWH in the ARMM and
out governmental functions. transfer of the admin and fiscal management of public works
vii) Abbas v COMELEC and funds to the ARG are meant to be true and unfettered.

(1) Thus, under the Constitution and R.A. No 6734, the creation of ix) Sema v COMELEC
the autonomous region shall take effect only when approved by (1) A province and a city cannot be created without a legislative
a majority of the votes cast by the constituent units in a district because it will violate the Constitution. That each city
plebiscite, and only those provinces and cities where a majority with a population of at least 250k or each province shall have at
vote in favor of the Organic Act shall be included in the least one representative. The power to create a province or city
autonomous region. The provinces and cities wherein such a inherently involves the power to create a legislative district.
majority is not attained shall not be included in the autonomous (2) Only Congress may increase the allowable membership in the
region. It may be that even if an autonomous region is created, House of Rep. Congress is also given the power to reapportion
not all of the thirteen (13) provinces and nine (9) cities legislative districts and create new legislative districts.
mentioned in Article II, section 1 (2) of R.A. No. 6734 shall be
included therein. The single plebiscite contemplated by the (3) Therefore, ARMM Regional Assembly cannot make a province,
Constitution and R.A. No. 6734 will therefore be determinative because making a province entails the creation of a new
of (1) whether there shall be an autonomous region in Muslim legislative district, and only Congress can make a legislative
district
Page 4 of 48
x) Cordillera Borad Coalition v COA designated by the President to undertake the Project. It follows
xi) Ordillo v COMELEC that the MMDA cannot validly order the elimination of
respondents’ terminals. MMDA is not vested with police power.
(1) The sole province of Ifugao cannot validly constitute Cordillera
Autonomous Region. The constitution requires more than one (2) The MMDA is devoid of authority to implement the Project as
LGU to be in the CAR. envisioned by the E.O; hence, it could not have been validly
designated by the President to undertake the Project. It follows
xii) MMDA v Bel-Air that the MMDA cannot validly order the elimination of
(1) MMDA is not a political unit of government. It does not have respondents’ terminals.
legislative powers. Its purpose is to formulate, coordinate,
regulate, implement, monitoring etc. It is merely administrative.
• SECTION 1. The territorial and political subdivisions of the Republic of the Philippines are
(2) Police power is delegated only to LGUs by virtue of the general the provinces, cities, municipalities, and barangays. There shall be autonomous regions in
welfare clause and is exercised by the LGUs through their Muslim Mindanao and the Cordilleras as hereinafter provided.
respective sanggunians.
• SECTION 2. The territorial and political subdivisions shall enjoy local autonomy.
(3) MMDA cannot exercise police power because it does not have
any delegated legislative power. Cannot enact ordinances. • SECTION 3. The Congress shall enact a local government code which shall provide for a
more responsive and accountable local government structure instituted through a system
xiii) MMDA v Garin of decentralization with effective mechanisms of recall, initiative, and referendum, allocate
(1) We concluded that the MMDA is not a local government unit or among the different local government units their powers, responsibilities, and resources,
a public corporation endowed with legislative power, and, unlike and provide for the qualifications, election, appointment and removal, term, salaries,
its predecessor, the Metro Manila Commission, it has no power powers and functions and duties of local officials, and all other matters relating to the
to enact ordinances for the welfare of the community. Thus, in organization and operation of the local units.
the absence of an ordinance from the City of Makati, its own • SECTION 4. The President of the Philippines shall exercise general supervision over local
order to open the street was invalid. governments. Provinces with respect to component cities and municipalities, and cities
(2) Having been lodged primarily in the National Legislature, it and municipalities with respect to component barangays shall ensure that the acts of their
cannot be exercised by any group or body of individuals not component units are within the scope of their prescribed powers and functions.
possessing legislative power. The National Legislature, • SECTION 5. Each local government unit shall have the power to create its own sources of
however, may delegate this power to the president and revenues and to levy taxes, fees, and charges subject to such guidelines and limitations as
administrative boards as well as the lawmaking bodies of the Congress may provide, consistent with the basic policy of local autonomy. Such taxes,
municipal corporations or local government units (LGUs). Once fees, and charges shall accrue exclusively to the local governments.
delegated, the agents can exercise only such legislative
powers as are conferred on them by the national lawmaking • SECTION 6. Local government units shall have a just share, as determined by law, in the
body. national taxes which shall be automatically released to them.
• SECTION 7. Local governments shall be entitled to an equitable share in the proceeds of
the utilization and development of the national wealth within their respective areas, in the
xiv)MMDA v Viron Transportation manner provided by law, including sharing the same with the inhabitants by way of direct
(1) the MMDA is devoid of authority to implement the Project as benefits.
envisioned by the E.O; hence, it could not have been validly
Page 5 of 48
• SECTION 8. The term of office of elective local officials, except barangay officials, which • SECTION 20. Within its territorial jurisdiction and subject to the provisions of this
shall be determined by law, shall be three years and no such official shall serve for more Constitution and national laws, the organic act of autonomous regions shall provide for
than three consecutive terms. Voluntary renunciation of the office for any length of time legislative powers over:
shall not be considered as an interruption in the continuity of his service for the full term for
• (1) Administrative organization;
which he was elected.
• (2) Creation of sources of revenues;
• SECTION 9. Legislative bodies of local governments shall have sectoral representation as
may be prescribed by law. • (3) Ancestral domain and natural resources;
• SECTION 10. No province, city, municipality, or barangay may be created, divided, • (4) Personal, family, and property relations;
merged, abolished, or its boundary substantially altered, except in accordance with the • (5) Regional urban and rural planning development;
criteria established in the Local Government Code and subject to approval by a majority of
the votes cast in a plebiscite in the political units directly affected. • (6) Economic, social, and tourism development;

• SECTION 15. There shall be created autonomous regions in Muslim Mindanao and in the • (7) Educational policies;
Cordilleras consisting of provinces, cities, municipalities, and geographical areas sharing • (8) Preservation and development of the cultural heritage; and
common and distinctive historical and cultural heritage, economic and social structures, • (9) Such other matters as may be authorized by law for the promotion of the general
and other relevant characteristics within the framework of this Constitution and the national welfare of the people of the region.
sovereignty as well as territorial integrity of the Republic of the Philippines.
• SECTION 21. The preservation of peace and order within the regions shall be the
• SECTION 16. The President shall exercise general supervision over autonomous regions responsibility of the local police agencies which shall be organized, maintained,
to ensure that the laws are faithfully executed. supervised, and utilized in accordance with applicable laws. The defense and security of
• SECTION 17. All powers, functions, and responsibilities not granted by this Constitution or the regions shall be the responsibility of the National Government.
by law to the autonomous regions shall be vested in the National Government. • Section 6. Authority to Create Local Government Units. - A local government unit may be
• SECTION 18. The Congress shall enact an organic act for each autonomous region with created, divided, merged, abolished, or its boundaries substantially altered either by law
the assistance and participation of the regional consultative commission composed of enacted by Congress in the case of a province, city, municipality, or any other political
representatives appointed by the President from a list of nominees from multisectoral subdivision, or by ordinance passed by the sangguniang panlalawigan or sangguniang
bodies. The organic act shall define the basic structure of government for the region panlungsod concerned in the case of a barangay located within its territorial jurisdiction,
consisting of the executive department and legislative assembly, both of which shall be subject to such limitations and requirements prescribed in this Code.
elective and representative of the constituent political units. The organic acts shall likewise • Section 7. Creation and Conversion. - As a general rule, the creation of a local
provide for special courts with personal, family, and property law jurisdiction consistent with government unit or its conversion from one level to another level shall be based on
the provisions of this Constitution and national laws. 
 verifiable indicators of viability and projected capacity to provide services, to wit:
The creation of the autonomous region shall be effective when approved by majority of the
votes cast by the constituent units in a plebiscite called for the purpose, provided that only • (a) Income. - It must be sufficient, based on acceptable standards, to provide for all
provinces, cities, and geographic areas voting favorably in such plebiscite shall be essential government facilities and services and special functions commensurate with
included in the autonomous region. the size of its population, as expected of the local government unit concerned;

• SECTION 19. The first Congress elected under this Constitution shall, within eighteen • (b) Population. - It shall be determined as the total number of inhabitants within the
months from the time of organization of both Houses, pass the organic acts for the territorial jurisdiction of the local government unit concerned; and
autonomous regions in Muslim Mindanao and the Cordilleras. • (c) Land Area. - It must be contiguous, unless it comprises two or more islands or is
separated by a local government unit independent of the others; properly identified by
Page 6 of 48
metes and bounds with technical descriptions; and sufficient to provide for such basic • Section 385. Manner of Creation. - A barangay may be created, divided, merged,
services and facilities to meet the requirements of its populace. abolished, or its boundary substantially altered, by law or by an ordinance of the
sangguniang panlalawigan or panlungsod, subject to approval by a majority of the votes
• Compliance with the foregoing indicators shall be attested to by the Department of
Finance (DOF), the National Statistics Office (NSO), and the Lands Management cast in a plebiscite to be conducted by the COMELEC in the local government unit or units
Bureau (LMB) of the Department of Environment and Natural Resources (DENR). directly affected within such period of time as may be determined by the law or ordinance
creating said barangay. In the case of the creation of barangays by the sangguniang
• Section 8. Division and Merger. - Division and merger of existing local government units panlalawigan, the recommendation of the sangguniang bayan concerned shall be
shall comply with the same requirements herein prescribed for their creation: Provided, necessary.
however, That such division shall not reduce the income, population, or land area of the
local government unit or units concerned to less than the minimum requirements • Section 386. Requisites for Creation. -
prescribed in this Code: Provided, further, That the income classification of the original • (a) A barangay may be created out of a contiguous territory which has a population of at
local government unit or units shall not fall below its current classification prior to such least two thousand (2,000) inhabitants as certified by the National Statistics Office
division.
 except in cities and municipalities within Metro Manila and other metropolitan political
The income classification of local government units shall be updated within six (6) months subdivisions or in highly urbanized cities where such territory shall have a certified
from the effectivity of this Code to reflect the changes in their financial position resulting population of at least five thousand (5,000) inhabitants: Provided, That the creation
from the increased revenues as provided herein. thereof shall not reduce the population of the original barangay or barangays to less
than the minimum requirement prescribed herein.

• Section 9. Abolition of Local Government Units. - A local government unit may be
abolished when its income, population, or land area has been irreversibly reduced to less To enhance the delivery of basic services in the indigenous cultural communities,
than the minimum standards prescribed for its creation under Book III of this Code, as barangays may be created in such communities by an Act of Congress, notwithstanding
certified by the national agencies mentioned in Section 7 hereof to Congress or to the the above requirement.
sangguniang concerned, as the case may be.
 • (b) The territorial jurisdiction of the new barangay shall be properly identified by metes
The law or ordinance abolishing a local government unit shall specify the province, city, and bounds or by more or less permanent natural boundaries. The territory need not be
municipality, or barangay with which the local government unit sought to be abolished will contiguous if it comprises two (2) or more islands.
be incorporated or merged.
• (c) The governor or city mayor may prepare a consolidation plan for barangays, based
• Section 10. Plebiscite Requirement. - No creation, division, merger, abolition, or on the criteria prescribed in this Section, within his territorial jurisdiction. The plan shall
substantial alteration of boundaries of local government units shall take effect unless be submitted to the sangguniang panlalawigan or sangguniang panlungsod concerned
approved by a majority of the votes cast in a plebiscite called for the purpose in the for appropriate action.

political unit or units directly affected. Said plebiscite shall be conducted by the In the case of municipalities within the Metropolitan Manila Area and other metropolitan
Commission on Elections (COMELEC) within one hundred twenty (120) days from the political subdivisions, the barangay consolidation plan shall be prepared and approved
date of effectivity of the law or ordinance effecting such action, unless said law or by the sangguniang bayan concerned.
ordinance fixes another date.
• Section 440. Role of the Municipality. - The municipality, consisting of a group of
• Section 384. Role of the Barangay. - As the basic political unit, the barangay serves as the barangays, serves primarily as a general purpose government for the coordination and
primary planning and implementing unit of government policies, plans, programs, projects, delivery of basic, regular and direct services and effective governance of the inhabitants
and activities in the community, and as a forum wherein the collective views of the people within its territorial jurisdiction.
may be expressed, crystallized and considered, and where disputes may be amicably
• Section 441. Manner of Creation. - A municipality may be created, divided, merged,
settled. abolished, or its boundary substantially altered only by an Act of Congress and subject to
the approval by a majority of the votes cast in a plebiscite to be conducted by the
Page 7 of 48
COMELEC in the local government unit or units directly affected. Except as may otherwise • Section 450. Requisites for Creation.
be provided in the said Act, the plebiscite shall be held within one hundred twenty (120)
• (a) A municipality or a cluster of barangays may be converted into a component city if it
days from the date of its effectivity. has an average annual income, as certified by the Department of Finance, of at least
• Section 442. Requisites for Creation. - Twenty million (P20,000,000.00) for the last two (2) consecutive years based on 1991
constant prices, and if it has either of the following requisites:
• (a) A municipality may be created if it has an average annual income, as certified by the
provincial treasurer, of at least Two million five hundred thousand pesos • (i) a contiguous territory of at least one hundred (100) square kilometers, as certified
(P2,500,000.00) for the last two (2) consecutive years based on the 1991 constant by the Lands Management Bureau; or
prices; a population of at least twenty-five thousand (25,000) inhabitants as certified by
• (ii) a population of not less than one hundred fifty thousand (150,000) inhabitants, as
the National Statistics Office; and a contiguous territory of at least fifty (50) square certified by the National Statistics Office:
kilometers as certified by the Lands Management Bureau: Provided, That the creation
thereof shall not reduce the land area, population or income of the original municipality • Provided, That, the creation thereof shall not reduce the land area, population, and
or municipalities at the time of said creation to less than the minimum requirements income of the original unit or units at the time of said creation to less than the
prescribed herein. minimum requirements prescribed herein.
• (b) The territorial jurisdiction of a newly-created municipality shall be properly identified • (b) The territorial jurisdiction of a newly-created city shall be properly identified by metes
by metes and bounds. The requirement on land area shall not apply where the and bounds. The requirement on land area shall not apply where the city proposed to be
municipality proposed to be created is composed of one (1) or more islands. The created is composed of one (1) or more islands. The territory need not be contiguous if it
territory need not be contiguous if it comprises two (2) or more islands. comprises two (2) or more islands.
• (c) The average annual income shall include the income accruing to the general fund of • (c) The average annual income shall include the income accruing to the general fund,
the municipality concerned, exclusive of special funds, transfers and non-recurring exclusive of specific funds, transfers, and non-recurring income.
income. • Section 451. Cities, Classified. - A city may either be component or highly urbanized:
• (d) Municipalities existing as of the date of the effectivity of this Code shall continue to Provided, however, That the criteria established in this Code shall not affect the
exist and operate as such. Existing municipal districts organized pursuant to presidential classification and corporate status of existing cities.

issuances or executive orders and which have their respective set of elective municipal Component cities whose charters prohibit their voters from voting for provincial elective
officials holding office at the time of the effectivity of this Code shall henceforth be officials. Independent component cities shall be independent of the province. Independent
considered as regular municipalities. component cities are those
• Section 448. Role of the City. - The city, consisting of more urbanized and developed • Section 452. Highly Urbanized Cities.
barangays. serves as a general purpose government for the coordination and delivery of • (a) Cities with a minimum population of two hundred thousand (200,000) inhabitants as
basic, regular, and direct services and effective governance of the inhabitants within its certified by the National Statistics Office, and within the latest annual income of at least
territorial jurisdiction. Fifty Million Pesos (P50,000,000.00) based on 1991 constant prices, as certified by the
• Section 449. Manner of Creation. - A city may be created, divided, merged, abolished, or city treasurer, shall be classified as highly urbanized cities.
its boundary substantially altered, only by an Act of Congress, and subject to approval by a • (b) Cities which do not meet above requirements shall be considered component cities
majority of the votes cast in a plebiscite to be conducted by the COMELEC in the local of the province in which they are geographically located. If a component city is located
government unit or units directly affected. Except as may otherwise be provided in such within the boundaries of two (2) or more provinces, such city shall be considered a
Act. the plebiscite shall be held within one hundred twenty (120) days from the date of its component of the province of which it used to be a municipality.
effectivity.

Page 8 of 48
• (c) Qualified voters of highly urbanized cities shall remain excluded from voting for • Section 462. Existing Sub-Provinces. - Existing sub-provinces are hereby converted into
elective provincial officials. regular provinces upon approval by a majority of the votes cast in a plebiscite to be held in
the said subprovinces and the original provinces directly affected. The plebiscite shall be
• Unless otherwise provided in the Constitution or this Code, qualified voters of
independent component cities shall be governed by their respective charters, as conducted by the COMELEC simultaneously with the national elections following the
amended, on the participation of voters in provincial elections. effectivity of this Code.

The new legislative districts created as a result of such conversion shall continue to be
• Qualified voters of cities who acquired the right to vote for elective provincial officials represented in Congress by the duly-elected representatives of the original districts out of
prior to the classification of said cities as highly-urbanized after the ratification of the which said new provinces or districts were created until their own representatives shall
Constitution and before the effectivity of this Code, shall continue to exercise such right. have been elected in the next regular congressional elections and qualified.

• Section 459. Role of the Province. - The province, composed of cluster of municipalities, The incumbent elected officials of the said subprovinces converted into regular provinces
or municipalities and component cities, and as a political and corporate unit of shall continue to hold office until June 30, 1992. Any vacancy occurring in the offices
government, serves as dynamic mechanism for developmental processes and effective occupied by said incumbent elected officials, or resulting from expiration of their terms of
governance of local government units within its territorial jurisdiction. office in case of a negative vote in the plebiscite results, shall be filled by appointment by
the President. The appointees shall hold office until their successors shall have been
• Section 460. Manner of Creation. - A province may be created, divided, merged, elected in the regular local elections following the plebiscite mentioned herein and
abolished, or its boundary substantially altered, only by an Act of Congress and subject to
qualified. After effectivity of such conversion, the President shall fill up the position of
approval by a majority of the votes cast in a plebiscite to be conducted by the COMELEC
governor of the newly-created province through appointment if none has yet been
in the local government unit or units directly affected. The plebiscite shall be held within
appointed to the same as hereinbefore provided, and shall also appoint a vice-governor
one hundred twenty (120) days from the date of effectivity of said Act, unless otherwise
and the other members of the sangguniang panlalawigan, all of whom shall likewise hold
provided therein.
office until their successors shall have been elected in the next regular local elections and
• Section 461. Requisites for Creation. qualified.

• (a) A province may be created if it has an average annual income, as certified by the All qualified appointive officials and employees in the career service of the said
Department of Finance, of not less than Twenty million pesos (P20,000,000.00) based subprovinces at the time of their conversion into regular provinces shall continue in office
on 1991 constant prices and either of the following requisites: in accordance with civil service law, rules and regulations.

• (i) a contiguous territory of at least two thousand (2,000) square kilometers, as


certified by the Lands Management Bureau; or (4) De Facto Municipal Corporations
• (ii) a population of not less than two hundred fifty thousand (250,000) inhabitants as (a) Municipality of San Narciso v Mendez
certified by the National Statistics Office:
i) Created in 1959 by virtue of Executive Order No. 353, the
• Provided, That, the creation thereof shall not reduce the land area, population, and Municipality of San Andres had been in existence for more than six
income of the original unit or units at the time of said creation to less than the years when, on 24 December 1965, Pelaez v. Auditor General was
minimum requirements prescribed herein. promulgated. The ruling could have sounded the call for a similar
• (b) The territory need not be contiguous if it comprise two (2) or more islands or is declaration of the unconstitutionality of Executive Order No. 353
separated by a chartered city or cities which do not contribute to the income of the but it was not to be the case. On the contrary, certain governmental
province. acts all pointed to the State's recognition of the continued
existence of the Municipality of San Andres. Thus, after more than
• (c) The average annual income shall include the income accruing to the general fund, five years as a municipal district, Executive Order No. 174
exclusive of special funds, trust funds, transfers and non-recurring income. classified the Municipality of San Andres as a fifth class
Page 9 of 48
municipality after having surpassed the income requirement laid a) An autonomous government that enjoys autonomy of the latter category
out in Republic Act No. 1515. Section 31 of Batas Pambansa Blg. [CONST. (1987), art. X, sec. 15.] is subject alone to the decree of the organic
129, otherwise known as the Judiciary Reorganization Act of 1980, act creating it and accepted principles on the effects and limits of "autonomy."
constituted as municipal circuits, in the establishment of Municipal On the other hand, an autonomous government of the former class is, as we
Circuit Trial Courts in the country, certain municipalities that noted, under the supervision of the national government acting through the
comprised the municipal circuits organized under Administrative President (and the Department of Local Government). 32 If the Sangguniang
Order No. 33, dated 13 June 1978, issued by this Court pursuant to Pampook (of Region XII), then, is autonomous in the latter sense, its acts are,
Presidential Decree No. 537. Under this administrative order, the debatably beyond the domain of this Court in perhaps the same way that the
Municipality of San Andres had been covered by the 10th Municipal internal acts, say, of the Congress of the Philippines are beyond our
Circuit Court of San Francisco-San Andres for the province of jurisdiction. But if it is autonomous in the former category only, it comes
Quezon. unarguably under our jurisdiction.
ii) Equally significant is Section 442(d) of the Local Government
Code to the effect that municipal districts "organized pursuant to 2. Pimentel Jr v Aguirre
presidential issuances or executive orders and which have their
respective sets of elective municipal officials holding office at the a) The Constitution vests the President with the power of supervision, not
time of the effectivity of (the) Code shall henceforth be considered control, over local government units (LGUs). Such power enables him to see
as regular municipalities.” to it that LGUs and their officials execute their tasks in accordance with law.
While he may issue advisories and seek their cooperation in solving economic
difficulties, he cannot prevent them from performing their tasks and using
(b) Camid v OP available resources to achieve their goals. He may not withhold or alter any
i) In the cases where a de facto municipal corporation was authority or power given them by the law. Thus, the withholding of a portion of
recognized as such despite the fact that the statute creating it was internal revenue allotments legally due them cannot be directed by
later invalidated, the decisions could fairly be made to rest on the administrative fiat.
consideration that there was some other valid law giving corporate
vitality to the organization. 3. LTO v Butuan
a) The devolution of the functions of the DOTC, performed by the LTFRB, to the
• (d) Municipalities existing as of the date of the effectivity of this Code shall continue to LGUs, as so aptly observed by the Solicitor General, is aimed at curbing the
exist and operate as such. Existing municipal districts organized pursuant to presidential alarming increase of accidents in national highways involving tricycles. It has
issuances or executive orders and which have their respective set of elective municipal been the perception that local governments are in good position to achieve the
officials holding office at the time of the effectivity of this Code shall henceforth be end desired by the law-making body because of their proximity to the situation
considered as regular municipalities. that can enable them to address that serious concern better than the national
government.

II. Principles of Local Autonomy b) It may not be amiss to state, nevertheless, that under Article 458 (a)[3-VI] of
the Local Government Code, the power of LGUs to regulate the operation of
A. Local Autonomy tricycles and to grant franchises for the operation thereof is still subject to the
1. Limbona v Mangelin guidelines prescribed by the DOTC. In compliance therewith, the Department
of Transportation and Communications ("DOTC") issued "Guidelines to
Page 10 of 48
Implement the Devolution of LTFRBs Franchising Authority over Tricycles-For- through a system of decentralization whereby local government units shall be given
Hire to Local Government units pursuant to the Local Government Code." more powers, authority, responsibilities, and resources. The process of decentralization
shall proceed from the national government to the local government units.

4. Drilon v Lim • (b) It is also the policy of the State to ensure the accountability of local government units
through the institution of effective mechanisms of recall, initiative and referendum.
a) An officer in control lays down the rules in the doing of an act. If they are not
followed, he may, in his discretion, order the act undone or re-done by his • (c) It is likewise the policy of the State to require all national agencies and offices to
subordinate or he may even decide to do it himself. Supervision does not conduct periodic consultations with appropriate local government units,
cover such authority. The supervisor or superintendent merely sees to it that nongovernmental and people's organizations, and other concerned sectors of the
the rules are followed, but he himself does not lay down such rules, nor does community before any project or program is implemented in their respective
he have the discretion to modify or replace them. If the rules are not observed, jurisdictions.1awphil.net
he may order the work done or re-done but only to conform to the prescribed • Section 3. Operative Principles of Decentralization. - The formulation and implementation
rules. He may not prescribe his own manner for the doing of the act. He has of policies and measures on local autonomy shall be guided by the following operative
no judgment on this matter except to see to it that the rules are followed. In principles:
the opinion of the Court, Secretary Drilon did precisely this, and no more nor
• (a) There shall be an effective allocation among the different local government units of
less than this, and so performed an act not of control but of mere supervision. their respective powers, functions, responsibilities, and resources;
• (b) There shall be established in every local government unit an accountable, efficient,
and dynamic organizational structure and operating mechanism that will meet the
priority needs and service requirements of its communities;
• SECTION 2. The territorial and political subdivisions shall enjoy local autonomy.
• SECTION 3. The Congress shall enact a local government code which shall provide for a • (c) Subject to civil service law, rules and regulations, local officials and employees paid
more responsive and accountable local government structure instituted through a system wholly or mainly from local funds shall be appointed or removed, according to merit and
of decentralization with effective mechanisms of recall, initiative, and referendum, allocate fitness, by the appropriate appointing authority;
among the different local government units their powers, responsibilities, and resources, • (d) The vesting of duty, responsibility, and accountability in local government units shall
and provide for the qualifications, election, appointment and removal, term, salaries, be accompanied with provision for reasonably adequate resources to discharge their
powers and functions and duties of local officials, and all other matters relating to the powers and effectively carry out their functions: hence, they shall have the power to
organization and operation of the local units. create and broaden their own sources of revenue and the right to a just share in national
taxes and an equitable share in the proceeds of the utilization and development of the
• SECTION 4. The President of the Philippines shall exercise general supervision over local
governments. Provinces with respect to component cities and municipalities, and cities national wealth within their respective areas;
and municipalities with respect to component barangays shall ensure that the acts of their • (e) Provinces with respect to component cities and municipalities, and cities and
component units are within the scope of their prescribed powers and functions. municipalities with respect to component barangays, shall ensure that the acts of their
component units are within the scope of their prescribed powers and functions;
• Section 2. Declaration of Policy. -
• (a) It is hereby declared the policy of the State that the territorial and political • (f) Local government units may group themselves, consolidate or coordinate their
subdivisions of the State shall enjoy genuine and meaningful local autonomy to enable efforts, services, and resources commonly beneficial to them;
them to attain their fullest development as self-reliant communities and make them more • (g) The capabilities of local government units, especially the municipalities and
effective partners in the attainment of national goals. Toward this end, the State shall barangays, shall be enhanced by providing them with opportunities to participate
provide for a more responsive and accountable local government structure instituted actively in the implementation of national programs and projects;
Page 11 of 48
• (h) There shall be a continuing mechanism to enhance local autonomy not only by • (d) Rights and obligations existing on the date of effectivity of this Code and arising out
legislative enabling acts but also by administrative and organizational reforms; of contracts or any other source of presentation involving a local government unit shall
be governed by the original terms and conditions of said contracts or the law in force at
• (i) Local government units shall share with the national government the responsibility in
the management and maintenance of ecological balance within their territorial the time such rights were vested; and
jurisdiction, subject to the provisions of this Code and national policies • (e) In the resolution of controversies arising under this Code where no legal provision or
jurisprudence applies, resort may be had to the customs and traditions in the place
• (j) Effective mechanisms for ensuring the accountability of local government units to
their respective constituents shall be strengthened in order to upgrade continually the where the controversies take place.
quality of local leadership; • Section 25. National Supervision over Local Government Units. -
• (k) The realization of local autonomy shall be facilitated through improved coordination • (a) Consistent with the basic policy on local autonomy, the President shall exercise
of national government policies and programs an extension of adequate technical and general supervision over local government units to ensure that their acts are within the
material assistance to less developed and deserving local government units scope of their prescribed powers and functions.

The President shall exercise supervisory authority directly over provinces, highly
• (l) The participation of the private sector in local governance, particularly in the delivery
of basic services, shall be encouraged to ensure the viability of local autonomy as an urbanized cities, and independent component cities; through the province with respect
alternative strategy for sustainable development; and to component cities and municipalities; and through the city and municipality with
respect to barangays.
• (m) The national government shall ensure that decentralization contributes to the
continuing improvement of the performance of local government units and the quality of • (b) National agencies and offices with project implementation functions shall coordinate
community life. with one another and with the local government units concerned in the discharge of
these functions. They shall ensure the participation of local government units both in the
• Section 4. Scope of Application. - This Code shall apply to all provinces, cities, planning and implementation of said national projects.
municipalities, barangays, and other political subdivisions as may be created by law, and,
to the extent herein provided, to officials, offices, or agencies of he national government. • (c) The President may, upon request of the local government unit concerned, direct the
appropriate national agency to provide financial, technical, or other forms of assistance
• Section 5. Rules of Interpretation. - In the interpretation of the provisions of this Code, the to the local government unit. Such assistance shall be extended at no extra cost to the
following rules shall apply: local government unit concerned.
• (a) Any provision on a power of a local government unit shall be liberally interpreted in • (d) National agencies and offices including government-owned or controlled
its favor, and in case of doubt, any question thereon shall be resolved in favor of corporations with field units or branches in a province, city, or municipality shall furnish
devolution of powers and of the lower local government unit. Any fair and reasonable the local chief executive concerned, for his information and guidance, monthly reports
doubt as to the existence of the power shall be interpreted in favor of the local including duly certified budgetary allocations and expenditures.
government unit concerned;
• Section 26. Duty of National Government Agencies in the Maintenance of Ecological
• (b) In case of doubt, any tax ordinance or revenue measure shall be construed strictly Balance. - It shall be the duty of every national agency or government-owned or controlled
against the local government unit enacting it, and liberally in favor of the taxpayer. Any corporation authorizing or involved in the planning and implementation of any project or
tax exemption, incentive or relief granted by any local government unit pursuant to the program that may cause pollution, climatic change, depletion of non-renewable resources,
provisions of this Code shall be construed strictly against the person claiming it. loss of crop land, rangeland, or forest cover, and extinction of animal or plant species, to
• (c) The general welfare provisions in this Code shall be liberally interpreted to give more consult with the local government units, nongovernmental organizations, and other sectors
powers to local government units in accelerating economic development and upgrading concerned and explain the goals and objectives of the project or program, its impact upon
the quality of life for the people in the community; the people and the community in terms of environmental or ecological balance, and the
measures that will be undertaken to prevent or minimize the adverse effects thereof.
Page 12 of 48
• Section 27. Prior Consultations Required. - No project or program shall be implemented a) Since, under Article X, Section 6 of the Constitution, only the just share of
by government authorities unless the consultations mentioned in Sections 2 (c) and 26 local governments is qualified by the words as determined by law, and not the
hereof are complied with, and prior approval of the sanggunian concerned is obtained: release thereof, the plain implication is that Congress is not authorized by the
Provided, That occupants in areas where such projects are to be implemented shall not be Constitution to hinder or impede the automatic release of the IRA.
evicted unless appropriate relocation sites have been provided, in accordance with the b) Indeed, that Article X, Section 6 of the Constitution did bind the legislative just
provisions of the Constitution. as much as the executive branch was presumed in the ruling of this Court in
the case of The Province of Batangas v. Romulo[17] which is analogous in
B. Creation of own sources of revenue and just share in national taxes many respects to the one at bar.

1. Batangas v Romulo
a) Section 4 of AO 372, however, orders the withholding, effective January 1, • SECTION 3.
1998, of 10 percent of the LGUs IRA pending the assessment and evaluation • SECTION 5.
by the Development Budget Coordinating Committee of the emerging fiscal
• SECTION 6. Local government units shall have a just share, as determined by law, in the
situation in the country. Such withholding clearly contravenes the Constitution national taxes which shall be automatically released to them.
and the law. Although temporary, it is equivalent to a holdback, which means
something held back or withheld, often temporarily. Hence, the temporary • SEC 1-5, LGC
nature of the retention by the national government does not matter. Any • Sec 284-294, LGC
retention is prohibited.
• Section 286. Automatic Release of Shares. -
b) The LGUs are not required to perform any act to receive the just share
accruing to them from the national coffers. As emphasized by the Local • (a) The share of each local government unit shall be released, without need of any
further action, directly to the provincial, city, municipal or barangay treasurer, as the
Government Code of 1991, the just share of the LGUs shall be released to
case may be, on a quarterly basis within five (5) days after the end of each quarter, and
them without need of further action.
which shall not be subject to any lien or holdback that may be imposed by the national
government for whatever purpose.
2. Quezon City v Bayantel • (b) Nothing in this Chapter shall be understood to diminish the share of local
a) In plain language, Section 11 of Rep. Act No. 7633 states that "the grantee, government units under existing laws.
its successors or assigns shall be liable to pay the same taxes on their real • Section 289. Share in the Proceeds from the Development and Utilization of the National
estate, buildings and personal property, exclusive of this franchise, as other Wealth. - Local government units shall have an equitable share in the proceeds derived
persons or corporations are now or hereafter may be required by law to pay." from the utilization and development of the national wealth within their respective areas,
The Court views this subsequent piece of legislation as an express and real including sharing the same with the inhabitants by way of direct benefits.
intention on the part of Congress to once again remove from the LGC’s
delegated taxing power, all of the franchisee’s (Bayantel’s) properties that are • Section 290. Amount of Share of Local Government Units. - Local government units shall,
in addition to the internal revenue allotment, have a share of forty percent (40%) of the
actually, directly and exclusively used in the pursuit of its franchise.
gross collection derived by the national government from the preceding fiscal year from
mining taxes, royalties, forestry and fishery charges, and such other taxes, fees, or
3. ACORD v Zamora charges, including related surcharges, interests, or fines, and from its share in any co-
production, joint venture or production sharing agreement in the utilization and
development of the national wealth within their territorial jurisdiction.
Page 13 of 48
• Section 291. Share of the Local Governments from any Government Agency or Owned or utilization and development of the national wealth, such share shall be directly remitted to
Controlled Corporation. - Local government units shall have a share based on the the provincial, city, municipal or barangay treasurer concerned within five (5) days after the
preceding fiscal year from the proceeds derived by any government agency or end of each quarter.
government-owned or controlled corporation engaged in the utilization and development of
• Section 294. Development and Livelihood Projects. - The proceeds from the share of local
the national wealth based on the following formula whichever will produce a higher share government units pursuant to this chapter shall be appropriated by their respective
for the local government unit: sanggunian to finance local government and livelihood projects: Provided, however, That
• (a) One percent (1%) of the gross sales or receipts of the preceding calendar year; or at least eighty percent (80%) of the proceeds derived from the development and utilization
of hydrothermal. geothermal, and other sources of energy shall be applied solely to lower
• (b) Forty percent (40%) of the mining taxes, royalties, forestry and fishery charges and
such other taxes, fees or charges, including related surcharges, interests, or fines the the cost of electricity in the local government unit where such a source of energy is
government agency or government owned or controlled corporation would have paid if it located.
were not otherwise exempt.
• Section 292. Allocation of Shares. - The share in the preceding Section shall be C. Allocate powers, responsibilities, and resources to LGUs
distributed in the following manner: 1. Atienza v Villarosa
• (a) Where the natural resources are located in the province: a) Under Rep. Act No. 7160, local legislative power for the province is exercised
• (1) Province - Twenty percent (20%); by the Sangguniang Panlalawigan[13] and the Vice-Governor is its presiding
officer.[14] Being vested with legislative powers, the Sangguniang
• (2) Component City/Municipality - Forty-five percent (45%); and
Panlalawigan enacts ordinances, resolutions and appropriates funds for the
• (3) Barangay - Thirty-five percent (35%) general welfare of the province in accordance with the provisions of Rep. Act
• Provided, however, That where the natural resources are located in two (2) or more No. 7160.[15] The same statute vests upon the Vice-Governor the power to:
provinces, or in two (2) or more component cities or municipalities or in two (2) or (1) (1) Be the presiding officer of the sangguniang panlalawigan and sign all
more barangays, their respective shares shall be computed on the basis of: warrants drawn on the provincial treasury for all expenditures appropriated
• (1) Population - Seventy percent (70%); and for the operation of the sangguniang panlalawigan.
• (2) Land area - Thirty percent (30%) (2) In case of temporary absence or incapacity of the department head or
chief of office, the officer next-in-rank shall automatically perform his
• (b) Where the natural resources are located in a highly urbanized or independent function and he shall be fully responsible therefor.
component city:
• (1) City - Sixty-five percent (65%); and
• (2) Barangay - Thirty-five percent (35%)
• Secs 3,7, Art X, 1987 Consti
• Provided, however, That where the natural resources are located in such two (2) or
more cities, the allocation of shares shall be based on the formula on population and • Secs 1-5, LGC
land area as specified in paragraph (a) of this Section.
• Section 293. Remittance of the Share of Local Government Units. - The share of local III. Powers of LGUs
government units from the utilization and development of national wealth shall be remitted
in accordance with Section 286 of this Code: Provided, however, That in the case of any A. Police Power(General Welfare Clause
government agency or government-owned or controlled corporation engaged in the 1. Tano v Socrates
Page 14 of 48
a) Finally, the centerpiece of LGC is the system of decentralization [26] as a) A third standard, denominated as heightened or immediate scrutiny, was later
expressly mandated by the Constitution. [27] Indispensable thereto is adopted by the U.S. Supreme Court for evaluating classifications based on
devolution and the LGC expressly provides that [a]ny provision on a power of gender53 and legitimacy.54 Immediate scrutiny was adopted by the U.S.
a local government unit shall be liberally interpreted in its favor, and in case of Supreme Court in Craig,55 after the Court declined to do so in Reed v. Reed.
doubt, any question thereon shall be resolved in favor of devolution of powers 56 While the test may have first been articulated in equal protection analysis,
and of the lower local government unit. Any fair and reasonable doubt as to it has in the United States since been applied in all substantive due process
the existence of the power shall be interpreted in favor of the local government cases as well.
unit concerned, [28] Devolution refers to the act by which the National b) We ourselves have often applied the rational basis test mainly in analysis of
Government confers power and authority upon the various local government equal protection challenges.57 Using the rational basis examination, laws or
units to perform specific functions and responsibilities. ordinances are upheld if they rationally further a legitimate governmental
b) interest.58 Under intermediate review, governmental interest is extensively
2. Batangas CATV v CA examined and the availability of less restrictive measures is considered.59
Applying strict scrutiny, the focus is on the presence of compelling, rather than
a) Where there is no express power in the charter of a municipality authorizing it substantial, governmental interest and on the absence of less restrictive
to adopt ordinances regulating certain matters which are specifically covered means for achieving that interest.
by a general statute, a municipal ordinance, insofar as it attempts to regulate
the subject which is completely covered by a general statute of the legislature, c) That the Ordinance prevents the lawful uses of a wash rate depriving patrons
may be rendered invalid. x x x Where the subject is of statewide concern, and of a product and the petitioners of lucrative business ties in with another
the legislature has appropriated the field and declared the rule, its declaration constitutional requisite for the legitimacy of the Ordinance as a police power
is binding throughout the State. measure. It must appear that the interests of the public generally, as
distinguished from those of a particular class, require an interference with
private rights and the means must be reasonably necessary for the
3. Magtajas v Pryce accomplishment of the purpose and not unduly oppressive of private rights.71
It must also be evident that no other alternative for the accomplishment of the
a) The tests of a valid ordinance are well established. A long line of decisions 9
purpose less intrusive of private rights can work. More importantly, a
has held that to be valid, an ordinance must conform to the following
reasonable relation must exist between the purposes of the measure and the
substantive requirements:
means employed for its accomplishment, for even under the guise of
(1) 1) It must not contravene the constitution or any statute. protecting the public interest, personal rights and those pertaining to private
(2) 2) It must not be unfair or oppressive. property will not be permitted to be arbitrarily invaded
(3) 3) It must not be partial or discriminatory.
(4) 4) It must not prohibit but may regulate trade. 5. SJS v Atienza, 2008

(5) 5) It must be general and consistent with public policy. a) Neither is it necessary to discuss at length the test of police power against the
assailed ordinance. Suffice it to state that the objective adopted by the
(6) 6) It must not be unreasonable. Sangguniang Panlungsod to promote the constituents’ general welfare in
terms of economic benefits cannot override the very basic rights to life,
security and safety of the people.
4. White Light Corp v City of Manila

Page 15 of 48
6. SJS v Atienza, 2014 a) The Sangguniang Panlalawigan's disapproval of Municipal Resolution No.
a) Essentially, the oil companies are fighting for their right to property. They 43-89 is an infirm action which does not render said resolution null and void.
allege that they stand to lose billions of pesos if forced to relocate. However, The law, as expressed in Section 153 of B.P. Blg. 337, grants the
based on the hierarchy of constitutionally protected rights, the right to life Sangguniang Panlalawigan the power to declare a municipal resolution invalid
enjoys precedence over the right to property. The reason is obvious: life is on the sole ground that it is beyond the power of the Sangguniang Bayan or
irreplaceable, property is not. When the state or LGU’s exercise of police the Mayor to issue.
power clashes with a few individuals’ right to property, the former should
prevail 3. Lagcao v Labra
b) a) The due process clause cannot be trampled upon each time an ordinance
orders the expropriation of a private individuals property. The courts cannot
even adopt a hands-off policy simply because public use or public purpose is
• Section 16. General Welfare. - Every local government unit shall exercise the powers
expressly granted, those necessarily implied therefrom, as well as powers necessary, invoked by an ordinance, or just compensation has been fixed and
appropriate, or incidental for its efficient and effective governance, and those which are determined.
essential to the promotion of the general welfare. Within their respective territorial b) The foundation of the right to exercise eminent domain is genuine necessity
jurisdictions, local government units shall ensure and support, among other things, the and that necessity must be of public character. [17] Government may not
preservation and enrichment of culture, promote health and safety, enhance the right of the capriciously or arbitrarily choose which private property should be
people to a balanced ecology, encourage and support the development of appropriate and expropriated. In this case, there was no showing at all why petitioners property
self-reliant scientific and technological capabilities, improve public morals, enhance was singled out for expropriation by the city ordinance or what necessity
economic prosperity and social justice, promote full employment among their residents, impelled the particular choice or selection. Ordinance No. 1843 stated no
maintain peace and order, and preserve the comfort and convenience of their inhabitants. reason for the choice of petitioners property as the site of a socialized housing
project.

B. Eminent Domain
1. Municipality of Paranaque v VM Realty Corp 4. City of Cebu v Spouses Dedamo

a) A local government unit (LGU), like the Municipality of Paraaque, cannot a) The petitioner has misread our ruling in The National Power Corp. vs. Court
authorize an expropriation of private property through a mere resolution of its of Appeals. [10] We did not categorically rule in that case that just
lawmaking body. The Local Government Code expressly and clearly requires compensation should be determined as of the filing of the complaint. We
an ordinance or a local law for the purpose. A resolution that merely expresses explicitly stated therein that although the general rule in determining just
the sentiment or opinion of the Municipal Council will not suffice. On the other compensation in eminent domain is the value of the property as of the date of
hand, the principle of res judicata does not bar subsequent proceedings for the filing of the complaint, the rule admits of an exception: where this Court
the expropriation of the same property when all the legal requirements for its fixed the value of the property as of the date it was taken and not at the date
valid exercise are complied with. of the commencement of the expropriation proceedings.

2. Moday v CA
• Section 19. Eminent Domain. - A local government unit may, through its chief executive
and acting pursuant to an ordinance, exercise the power of eminent domain for public use,
Page 16 of 48
or purpose or welfare for the benefit of the poor and the landless, upon payment of just
compensation, pursuant to the provisions of the Constitution and pertinent laws: Provided, 3. Smart Communications v City of Davao
however, That the power of eminent domain may not be exercised unless a valid and
definite offer has been previously made to the owner, and such offer was not accepted: a) The in lieu of all taxes clause applies only to national internal revenue taxes
Provided, further, That the local government unit may immediately take possession of the and not to local taxes. [T]he "in lieu of all taxes" clause in Smart's franchise
property upon the filing of the expropriation proceedings and upon making a deposit with refers only to taxes, other than income tax, imposed under the National
the proper court of at least fifteen percent (15%) of the fair market value of the property Internal Revenue Code. The "in lieu of all taxes" clause does not apply to local
based on the current tax declaration of the property to be expropriated: Provided, finally, taxes. The proviso in the first paragraph of Section 9 of Smart's franchise
That, the amount to be paid for the expropriated property shall be determined by the states that the grantee shall "continue to be liable for income taxes payable
proper court, based on the fair market value at the time of the taking of the property. under Title II of the National Internal Revenue Code." Also, the second
paragraph of Section 9 speaks of tax returns filed and taxes paid to the
"Commissioner of Internal Revenue or his duly authorized representative in
C. Taxing Power accordance with the National Internal Revenue Code." Moreover, the same
1. Mactan-Cebu International Airport Authority v City of Lapu-Lapu paragraph declares that the tax returns "shall be subject to audit by the
Bureau of Internal Revenue." Nothing is mentioned in Section 9 about local
a) The petitioner is an instrumentality of the government; thus, its properties taxes. The clear intent is for the "in lieu of all taxes" clause to apply only to
actually, solely and exclusively used for public purposes, consisting of the taxes under the National Internal Revenue Code and not to local taxes. The
airport terminal building, airfield, runway, taxiway and the lots on which they only interpretation, under the rule on strict construction of tax exemptions, is
are situated, are not subject to real property tax and respondent City is not that the "in lieu of all taxes" clause in Smart's franchise refers only to national
justified in collecting taxes from petitioner over said properties. and not to local taxes.

2. Pelizloy Realty Corp v Benguet Province • Sec 5, Art X, 1987 Consti


a) Evidently, Section 140 of the LGC carves a clear exception to the general • Section 18. Power to Generate and Apply Resources. - Local government units shall
rule in Section 133 (i). Section 140 expressly allows for the imposition by have the power and authority to establish an organization that shall be responsible for the
provinces of amusement ta xes on “the proprietors, lessees, or operators of efficient and effective implementation of their development plans, program objectives and
theaters, cinemas, conc ert halls, circuses, boxing stadia, and other places of priorities; to create their own sources of revenues and to levy taxes, fees, and charges
amusement.” However, resorts, swimming pools, bath houses, hot springs, which shall accrue exclusively for their use and disposition and which shall be retained by
and tourist spots are not among those places expressly mentioned by Section them; to have a just share in national taxes which shall be automatically and directly
140 of the LGC as being subject to amusem ent taxes. Thus, the released to them without need of any further action; to have an equitable share in the
determination of whether amusement taxes may be levied on admissions to proceeds from the utilization and development of the national wealth and resources within
resorts, swimming pools, bath houses, hot springs, and tourist spots hinges their respective territorial jurisdictions including sharing the same with the inhabitants by
on whether the phrase ‘other places of amusement’ encompasses resorts, way of direct benefits; to acquire, develop, lease, encumber, alienate, or otherwise dispose
swimming pools, bath houses, hot springs, and tourist spots. Under the of real or personal property held by them in their proprietary capacity and to apply their
principle of ejusdem generis , “where a gene ral word or phrase follows an resources and assets for productive, developmental, or welfare purposes, in the exercise
enumeration of particular and specific words of the same class or where the or furtherance of their governmental or proprietary powers and functions and thereby
latter follow the form er, the general word or phrase is to be construed to ensure their development into self-reliant communities and active participants in the
include, or to be restrict ed to persons, things or cases akin to, resembling, or attainment of national goals.
of the same kind or cl ass as those specifically mentioned.”
Page 17 of 48
• Section 128. Scope. - The provisions herein shall govern the exercise by provinces, • (d) Customs duties, registration fees of vessel and wharfage on wharves, tonnage dues,
cities, municipalities, and barangays of their taxing and other revenue-raising powers. and all other kinds of customs fees, charges and dues except wharfage on wharves
constructed and maintained by the local government unit concerned;
• Section 129. Power to Create Sources of Revenue. - Each local government unit shall
exercise its power to create its own sources of revenue and to levy taxes, fees, and • (e) Taxes, fees, and charges and other impositions upon goods carried into or out of, or
charges subject to the provisions herein, consistent with the basic policy of local autonomy. passing through, the territorial jurisdictions of local government units in the guise of
Such taxes, fees, and charges shall accrue exclusively to the local government units. charges for wharfage, tolls for bridges or otherwise, or other taxes, fees, or charges in
any form whatsoever upon such goods or merchandise;
• Section 130. Fundamental Principles. - The following fundamental principles shall govern
the exercise of the taxing and other revenue-raising powers of local government units: • (f) Taxes, fees or charges on agricultural and aquatic products when sold by marginal
farmers or fishermen;
• (a) Taxation shall be uniform in each local government unit;
• (b) Taxes, fees, charges and other impositions shall: • (g) Taxes on business enterprises certified to by the Board of Investments as pioneer or
non-pioneer for a period of six (6) and four (4) years, respectively from the date of
• (1) be equitable and based as far as practicable on the taxpayer's ability to pay; registration;
• (2) be levied and collected only for public purposes; • (h) Excise taxes on articles enumerated under the national Internal Revenue Code, as
• (3) not be unjust, excessive, oppressive, or confiscatory; amended, and taxes, fees or charges on petroleum products;
• (4) not be contrary to law, public policy, national economic policy, or in the restraint of • (i) Percentage or value-added tax (VAT) on sales, barters or exchanges or similar
trade; transactions on goods or services except as otherwise provided herein;
• (c) The collection of local taxes, fees, charges and other impositions shall in no case be • (j) Taxes on the gross receipts of transportation contractors and persons engaged in the
let to any private person; transportation of passengers or freight by hire and common carriers by air, land or water,
except as provided in this Code;
• (d) The revenue collected pursuant to the provisions of this Code shall inure solely to
the benefit of, and be subject to the disposition by, the local government unit levying the • (k) Taxes on premiums paid by way or reinsurance or retrocession;
tax, fee, charge or other imposition unless otherwise specifically provided herein; and, • (l) Taxes, fees or charges for the registration of motor vehicles and for the issuance of all
• (e) Each local government unit shall, as far as practicable, evolve a progressive system kinds of licenses or permits for the driving thereof, except tricycles;
of taxation. • (m) Taxes, fees, or other charges on Philippine products actually exported, except as
• Section 132. Local Taxing Authority. - The power to impose a tax, fee, or charge or to otherwise provided herein;
generate revenue under this Code shall be exercised by the sanggunian of the local • (n) Taxes, fees, or charges, on Countryside and Barangay Business Enterprises and
government unit concerned through an appropriate ordinance. cooperatives duly registered under R.A. No. 6810 and Republic Act Numbered Sixty-
• Section 133. Common Limitations on the Taxing Powers of Local Government Units. - nine hundred thirty-eight (R.A. No. 6938) otherwise known as the "Cooperative Code of
Unless otherwise provided herein, the exercise of the taxing powers of provinces, cities, the Philippines" respectively; and
municipalities, and barangays shall not extend to the levy of the following: • (o) Taxes, fees or charges of any kind on the National Government, its agencies and
• (a) Income tax, except when levied on banks and other financial institutions; instrumentalities, and local government units.
• (b) Documentary stamp tax;
• (c) Taxes on estates, inheritance, gifts, legacies and other acquisitions mortis causa, D. Closure and opening of roads
except as otherwise provided herein;

Page 18 of 48
• Section 21. Closure and Opening of Roads. - 1. Requisites for valid ordinance (GCUUPP)
• (a) A local government unit may, pursuant to an ordinance, permanently or a) Must not be contrary to constitution or a statute
temporarily close or open any local road, alley, park, or square falling within its b) not unfair or oppressive
jurisdiction: Provided, however, That in case of permanent closure, such ordinance
must be approved by at least two-thirds (2/3) of all the members of the sanggunian, and c) not partial or discriminatory
when necessary, an adequate substitute for the public facility that is subject to closure is d) not prohibit but must regulate free trade
provided.
e) general and consistent with public policy
• (b) No such way or place or any part thereof shall be permanently closed without f) not unreasonable
making provisions for the maintenance of public safety therein. A property thus
permanently withdrawn from public use may be used or conveyed for any purpose for 2. Local Initiative and Referendum
which other real property belonging to the local government unit concerned may be a) Tatel v Virac
lawfully used or conveyed: Provided, however, That no freedom park shall be closed
permanently without provision for its transfer or relocation to a new site. (1) ——

• (c) Any national or local road, alley, park, or square may be temporarily closed during an b) Tan v Perea
actual emergency, or fiesta celebrations, public rallies, agricultural or industrial fairs, or (1) Ordinance must not be against the law
an undertaking of public works and highways, telecommunications, and waterworks
c) Malonzo v Zamora
projects, the duration of which shall be specified by the local chief executive concerned
in a written order: Provided, however, That no national or local road, alley, park, or (1) The sanggunian shall adopt or update its existing rules of at the start of the
square shall be temporarily closed for athletic, cultural, or civic activities not officially first regular session
sponsored, recognized, or approved by the local government unit concerned. d) La Carlota v Roko
• (d) Any city, municipality, or barangay may, by a duly enacted ordinance, temporarily (1) Vice Mayor is presiding officer of Sanggunian therefore he is a member.
close and regulate the use of any local street, road, thoroughfare, or any other public However he shall only vote to break a tie.
place where shopping malls, Sunday, flea or night markets, or shopping areas may be
established and where goods, merchandise, foodstuffs, commodities, or articles of (2) Vice Mayor is necessarily part of the numbering of quorum
commerce may be sold and dispensed to the general public. e) SBMA v COMELEC
• NOTES: (1) The Constitution is clearly includes no only ordinance but resolutions as
appropriate subjects of initiative and referendum
• Ordinance
• Permanent – 2/3 of all the members of the sanggunian and when necessary, an
adequate substitute for the public facility that is subject to closure is provided. • Section 48. Local Legislative Power. - Local legislative power shall be exercised by the
sangguniang panlalawigan for the province; the sangguniang panlungsod for the city; the
• There must be a provision for the maintenance of public safety.
sangguniang bayan for the municipality; and the sangguniang barangay for the barangay.
• Those permanently withdrawn from public use
• Q – Is section 48 an absolute rule?
• Temporary – Majority of all the members
• A – NO! People themselves may exercise legislative power because there is initiative and
referendum
E. Legislative Power • Section 49. Presiding Officer. -
Page 19 of 48
• (a) The vice-governor shall be the presiding officer of the sangguniang panlalawigan; • (a) Every sanggunian member shall, upon assumption to office, make a full disclosure of
the city vice-mayor, of the sangguniang panlungsod; the municipal vice-mayor, of the his business and financial interests, or professional relationship or any relation by affinity
sangguniang bayan; and the punong barangay, of the sangguniang barangay. The or consanguinity within the fourth civil degree, which he may have with any person, firm,
presiding officer shall vote only to break a tie. or entity affected by any ordinance or resolution under consideration by the sanggunian
of which he is a member, which relationship may result in conflict of interest. Such
• (b) In the event of the inability of the regular presiding officer to preside at a sanggunian
session, the members present and constituting a quorum shall elect from among relationship shall include:
themselves a temporary presiding officer. He shall certify within ten (10) days from the • (1) Ownership of stock or capital, or investment, in the entity or firm to which the
passage of ordinances enacted and resolutions adopted by the sanggunian in the ordinance or resolution may apply; and
session over which he temporarily presided.
• (2) Contracts or agreements with any person or entity which the ordinance or
• Section 50. Internal Rules of Procedure. - resolution under consideration may affect.
• (a) On the first regular session following the election of its members and within ninety • In the absence of a specific constitutional or statutory provision applicable to this
(90) days thereafter, the sanggunian concerned shall adopt or update its existing rules situation, "conflict of interest" refers in general to one where it may be reasonably
of procedure. deduced that a member of a sanggunian may not act in the public interest due to
some private, pecuniary, or other personal considerations that may tend to affect his
• (b) The rules of procedure shall provided for the following:
judgment to the prejudice of the service or the public.
• (1) The organization of the sanggunian and the election of its officers as well as the
creation of standing committees which shall include, but shall not be limited to, the • (b) The disclosure required under this Act shall be made in writing and submitted to the
committees on appropriations, women and family, human rights, youth and sports secretary of the sanggunian or the secretary of the committee of which he is a member.
development, environmental protection, and cooperatives; the general jurisdiction of The disclosure shall, in all cases, form part of the record of the proceedings and shall be
each committee; and the election of the chairman and members of each committee; made in the following manner:

• (2) The order and calendar of business for each session; • (1) Disclosure shall be made before the member participates in the deliberations on
the ordinance or resolution under consideration: Provided, That, if the member did not
• (3) The legislative process; participate during the deliberations, the disclosure shall be made before voting on the
• (4) The parliamentary procedures which include the conduct of members during ordinance or resolution on second and third readings; and
sessions; • (2) Disclosure shall be made when a member takes a position or makes a privilege
• (5) The discipline of members for disorderly behavior and absences without justifiable speech on a matter that may affect the business interest, financial connection, or
cause for four (4) consecutive sessions, for which they may be censured, professional relationship described herein.
reprimanded, or excluded from the session, suspended for not more than sixty (60) • Section 52. Sessions. -
days, or expelled: Provided, That the penalty of suspension or expulsion shall require
the concurrence of at least two-thirds (2/3) vote of all the sanggunian members: • (a) On the first day of the session immediately following the election of its members, the
Provided, further, That a member convicted by final judgment to imprisonment of at sanggunian shall, by resolution, fix the day, time, and place of its regular sessions. The
least one (1) year for any crime involving moral turpitude shall be automatically minimum numbers of regular sessions shall be once a week for the sangguniang
expelled from the sanggunian; and panlalawigan, sangguniang panlungsod, and sangguniang bayan, and twice a month for
the sangguniang barangay.
• (6) Such other rules as the sanggunian may adopt.
• (b) When public interest so demands, special sessions may be called by the local chief
lawphil™

• Section 51. Full Disclosure of Financial and Business Interests of Sanggunian Members. - executive or by a majority of the members of the sanggunian.

Page 20 of 48
• (c) All sanggunian sessions shall be open to the public unless a closed-door session is duly approved by the members present, shall then declare the session adjourned for
ordered by an affirmative vote of a majority of the members present, there being a lack of quorum.
quorum, in the public interest or for reasons of security, decency, or morality. No two (2)
• Section 54. Approval of Ordinances. -
sessions, regular or special, may be held in a single day.
• (a) Every ordinance enacted by the sangguniang panlalawigan, sangguniang
• (d) In the case of special sessions of the sanggunian, a written notice to the members panlungsod, or sangguniang bayan shall be presented to the provincial governor or city
shall be served personally at the member's usual place of residence at least twenty-four or municipal mayor, as the case may be. If the local chief executive concerned approves
(24) hours before the special session is held.
 the same, he shall affix his signature on each and every page thereof; otherwise, he
Unless otherwise concurred in by two-thirds (2/3) vote of the sanggunian members shall veto it and return the same with his objections to the sanggunian, which may
present, there being a quorum, no other matters may be considered at a special session proceed to reconsider the same. The sanggunian concerned may override the veto of
except those stated in the notice. the local chief executive by two-thirds (2/3) vote of all its members, thereby making the
• (e) Each sanggunian shall keep a journal and record of its proceedings which may be ordinance or resolution effective for all legal intents and purposes.
published upon resolution of the sanggunian concerned.
• (b) The veto shall be communicated by the local chief executive concerned to the
• NOTES sanggunian within fifteen (15) days in the case of a province, and ten (10) days in the
case of a city or a municipality; otherwise, the ordinance shall be deemed approved as if
• Recess v Adjournment
he had signed it.
• Adjournment – congress is not in session; basta natapos mo ung agenda,
adjournment. It ends the legislative day • (c) Ordinances enacted by the sangguniang barangay shall, upon approval by the
majority of all its members, be signed by the punong barangay.
• Recess – hindi natapos ung agend pero umuwi na, un ung recess. Continuation of
legislative day. • Section 55. Veto Power of the Local Chief Executive. -
• No two (2) sessions, regular or special, may be held in a single day. • (a) The local chief executive may veto any ordinance of the sanggunian panlalawigan,
sangguniang panlungsod, or sanggunian bayan on the ground that it is ultra vires or
prejudicial to the public welfare, stating his reasons therefor in writing.
• Section 53. Quorum. - • (b) The local chief executive, except the punong barangay, shall have the power to veto
• (a) A majority of all the members of the sanggunian who have been elected and qualified any particular item or items of an appropriations ordinance, an ordinance or resolution
shall constitute a quorum to transact official business. Should a question of quorum be adopting a local development plan and public investment program, or an ordinance
raised during a session, the presiding officer shall immediately proceed to call the roll of directing the payment of money or creating liability. In such a case, the veto shall not
the members and thereafter announce the results. affect the item or items which are not objected to. The vetoed item or items shall not
take effect unless the sanggunian overrides the veto in the manner herein provided;
• (b) Where there is no quorum, the presiding officer may declare a recess until such time otherwise, the item or items in the appropriations ordinance of the previous year
as a quorum is constituted, or a majority of the members present may adjourn from day
corresponding to those vetoed, if any, shall be deemed reenacted.
to day and may compel the immediate attendance of any member absent without
justifiable cause by designating a member of the sanggunian to be assisted by a • (c) The local chief executive may veto an ordinance or resolution only once. The
member or members of the police force assigned in the territorial jurisdiction of the local sanggunian may override the veto of the local chief executive concerned by two-thirds
government unit concerned, to arrest the absent member and present him at the (2/3) vote of all its members, thereby making the ordinance effective even without the
session. approval of the local chief executive concerned.
• (c) If there is still no quorum despite the enforcement of the immediately preceding • Section 56. Review of Component City and Municipal Ordinances or Resolutions by the
subsection, no business shall be transacted. The presiding officer, upon proper motion Sangguniang Panlalawigan.
Page 21 of 48
• (a) Within three (3) days after approval, the secretary to the sanggunian panlungsod or • Section 58. Enforcement of Disapproved Ordinances or Resolutions. - Any attempt to
sangguniang bayan shall forward to the sangguniang panlalawigan for review, copies of enforce any ordinance or any resolution approving the local development plan and public
approved ordinances and the resolutions approving the local development plans and investment program, after the disapproval thereof, shall be sufficient ground for the
public investment programs formulated by the local development councils. suspension or dismissal of the official or employee concerned.
• (b) Within thirty (30) days after the receipt of copies of such ordinances and resolutions, • Section 59. Effectivity of Ordinances or Resolutions. -
the sangguniang panlalawigan shall examine the documents or transmit them to the
• (a) Unless otherwise stated in the ordinance or the resolution approving the local
provincial attorney, or if there be none, to the provincial prosecutor for prompt development plan and public investment program, the same shall take effect after ten
examination. The provincial attorney or provincial prosecutor shall, within a period of ten (10) days from the date a copy thereof is posted in a bulletin board at the entrance of
(10) days from receipt of the documents, inform the sangguniang panlalawigan in writing the provincial capitol or city, municipal, or barangay hall, as the case may be, and in at
of his comments or recommendations, which may be considered by the sangguniang least two (2) other conspicuous places in the local government unit concerned.
panlalawigan in making its decision.
• (b) The secretary to the sanggunian concerned shall cause the posting of an ordinance
• (c) If the sangguniang panlalawigan finds that such an ordinance or resolution is beyond or resolution in the bulletin board at the entrance of the provincial capitol and the city,
the power conferred upon the sangguniang panlungsod or sangguniang bayan municipal, or barangay hall in at least two (2) conspicuous places in the local
concerned, it shall declare such ordinance or resolution invalid in whole or in part. The government unit concerned not later than five (5) days after approval thereof.

sangguniang panlalawigan shall enter its action in the minutes and shall advise the The text of the ordinance or resolution shall be disseminated and posted in Filipino or
corresponding city or municipal authorities of the action it has taken. English and in the language understood by the majority of the people in the local
• (d) If no action has been taken by the sangguniang panlalawigan within thirty (30) days government unit concerned, and the secretary to the sanggunian shall record such fact
after submission of such an ordinance or resolution, the same shall be presumed in a book kept for the purpose, stating the dates of approval and posting.
consistent with law and therefore valid.
• (c) The gist of all ordinances with penal sanctions shall be published in a newspaper of
• Section 57. Review of Barangay Ordinances by the Sangguniang Panlungsod or general circulation within the province where the local legislative body concerned
Sangguniang Bayan. - belongs. In the absence of any newspaper of general circulation within the province,
posting of such ordinances shall be made in all municipalities and cities of the province
• (a) Within ten (10) days after its enactment, the sangguniang barangay shall furnish
copies of all barangay ordinances to the sangguniang panlungsod or sangguniang where the sanggunian of origin is situated.
bayan concerned for review as to whether the ordinance is consistent with law and city • (d) In the case of highly urbanized and independent component cities, the main features
or municipal ordinances. of the ordinance or resolution duly enacted or adopted shall, in addition to being posted,
be published once in a local newspaper of general circulation within the city: Provided,
• (b) If the sangguniang panlungsod or sangguniang bayan, as the case may be, fails to
take action on barangay ordinances within thirty (30) days from receipt thereof, the That in the absence thereof the ordinance or resolution shall be published in any
same shall be deemed approved. newspaper of general circulation.

• (c) If the sangguniang panlungsod or sangguniang bayan, as the case may be, finds the • Q – May a resolution be a subject to initiative/referendum
barangay ordinances inconsistent with law or city or municipal ordinances, the • A – YES! RA 6735, see SBMA v. COMELEC. The Constitution clearly includes not only
sanggunian concerned shall, within thirty (30) days from receipt thereof, return the same ordinances but resolutions as appropriate subjects of local initiative.
with its comments and recommendations to the sangguniang barangay concerned for
adjustment, amendment, or modification; in which case, the effectivity of the barangay
ordinance is suspended until such time as the revision called for is effected. F. Corporate Powers
1. To sue and be sued

Page 22 of 48
a) Suability v Liability iii) When the interests of a provincial government and of any political
(1) A distinction should first be made between suability and liability. division thereof are opposed, the provincial fiscal shall act on
behalf of the province.
(a) "Suability depends on the consent of the state to be sued, liability on
the applicable law and the established facts. The circumstance that a iv) When the provincial fiscal is disqualified to serve any municipality
state is suable does not necessarily mean that it is liable; on the other or other political subdivision of a province, a special attorney may
hand, it can never be held liable if it does not first consent to be sued. be employed by its council.

(b) Liability is not conceded by the mere fact that the state has allowed (b) As already stated, private lawyers may not represent municipalities on
itself to be sued. When the state does waive its sovereign immunity, it their own. Neither may they do so even in collaboration with
is only giving the plaintiff the chance to prove, if it can, that the authorized government lawyers. This is anchored on the principle
defendant is liable. (Municipality of San Fernando, La Union v. that only accountable public officers may act for and in behalf of public
Firme) entities and that public funds should not be expended to hire private
lawyers.
i) the test of liability of the municipality depends on whether or not the
driver, acting in behalf of the municipality, is performing c) Province of Zamboanga v City of Zamboanga
governmental or proprietary functions. As emphasized in the (1) For purpose of this article, the principles, obtaining under the Law of
case of Torio vs. Fontanilla (G. R. No. L-29993, October 23, 1978. Municipal Corporations can be considered as "special laws". Hence, the
85 SCRA 599, 606), the distinction of powers becomes important classification of municipal property devoted for distinctly governmental
for purposes of determining the liability of the municipality for the purposes as public should prevail over the Civil Code classification in this
acts of its agents which result in an injury to third persons. particular case.
2. To acquire and sell property (2) to be considered public, it is enough that the property be held and,
3. To enter into contracts devoted for governmental purposes like local administration, public
education, public health, etc.
a) Munic of San Fernando v Firme
(3) If the property is owned by the municipality (meaning municipal
(1) Suability depends on the consent of the State to be sued. Although it is corporation) in its public and governmental capacity, the property is public
suable, it doesnt mean that its liable. and Congress has absolute control over it. But if the property is owned in
b) Ramos v CA its private or proprietary capacity, then it is patrimonial and Congress has
no absolute control. The municipality cannot be deprived of it without due
(1) Who may appear in behalf of the LGU?
process and payment of just compensation.
(a) The provincial fiscal shall represent the province and any municipality
or municipal district thereof in any court, except
d) Quezon v Lexber
i) in cases whereof (sic) original jurisdiction is vested in the Supreme
Court or (1) Unless otherwise provided in this Code, no contract may be entered into
by the local chief executive in behalf of the local government unit without
ii) in cases where the municipality or municipal district in question is a
prior authorization by the sanggunian concerned. A legible copy of such
party adverse to the provincial government or to some other
contract shall be posted at a conspicuous place in the provincial capitol or
municipality or municipal district in the same province.
the city, municipal or barangay hall.

Page 23 of 48
• Section 22. Corporate Powers. - a) Lozano was employed as a driver as the municipality. The mayor is not the
employer. Employer-employee relationship exists even if the employee is
• (a) Every local government unit, as a corporation, shall have the following powers:
loaned.
• (1) To have continuous succession in its corporate name;
b) It was the Municipality of Koronadal which was the lawful employer of Lozano
• (2) To sue and be sued; at the time of the accident. It is uncontested that Lozano was employed as a
• (3) To have and use a corporate seal; driver by the municipality. That he was subsequently assigned to Mayor
Miguel during the time of the accident is of no moment. This Court has, on
• (4) To acquire and convey real or personal property; several occasions, held that an employer-employee relationship still exists
• (5) To enter into contracts; and even if the employee was loaned by the employer to another person or entity
• (6) To exercise such other powers as are granted to corporations, subject to the because control over the employee subsists. In the case under review,
limitations provided in this Code and other laws. the Municipality of Koronadal remains to be Lozanos employer
notwithstanding Lozanos assignment to Mayor Miguel.
• (b) Local government units may continue using, modify, or change their existing
corporate seals: Provided, That newly established local government units or those
without corporate seals may create their own corporate seals which shall be registered • Section 24. Liability for Damages. - Local government units and their officials are not
with the Department of the Interior and Local Government: Provided, further, That any exempt from liability for death or injury to persons or damage to property.
change of corporate seal shall also be registered as provided hereon.
• Art. 2180. The obligation imposed by Article 2176 is demandable not only for one's own
• (c) Unless otherwise provided in this Code, no contract may be entered into by the local acts or omissions, but also for those of persons for whom one is responsible. 

chief executive in behalf of the local government unit without prior authorization by the The State is responsible in like manner when it acts through a special agent; but not when
sanggunian concerned. A legible copy of such contract shall be posted at a conspicuous the damage has been caused by the official to whom the task done properly pertains, in
place in the provincial capitol or the city, municipal or barangay hall. which case what is provided in Article 2176 shall be applicable.
• (d) Local government units shall enjoy full autonomy in the exercise of their proprietary • Art. 2189. Provinces, cities and municipalities shall be liable for damages for the death of,
functions and in the limitations provided in this Code and other applicable laws, or injuries suffered by, any person by reason of the defective condition of roads, streets,
bridges, public buildings, and other public works under their control or supervision. (n)
G. Liability of LGUs
1. Municipality of San Juan v CA H. Settlement of boundary disputes

a) Although it is a national road, as long is it is under their control or supervision, 1. Pasig v COMELEC
the LGU is liable. a) Pending boundary dispute case between two local government units presents
(1) For liability to arise under Article 2189 of the Civil Code, ownership of the a prejudicial question which must be decided before plebiscite for creation
roads, streets, bridges, public buildings and other public works, is not a of barangays may be held.
controlling factor, it being sufficient that a province, city or municipality has b) In the case at bar, while the City of Pasig vigorously claims that the areas
control or supervision thereof. covered by the proposed Barangays Karangalan and Napico are within its
territory, it can not deny that portions of the same area are included in the
boundary dispute case pending before the Regional Trial Court of Antipolo.
2. Jayme v Apostol Surely, whether the areas in controversy shall be decided as within the
territorial jurisdiction of the Municipality of Cainta or the City of Pasig has
Page 24 of 48
material bearing to the creation of the proposed Barangays Karangalan and • (a) Boundary disputes involving two (2) or more barangays in the same city or
Napico. Indeed, a requisite for the creation of a barangay is for its territorial municipality shall be referred for settlement to the sangguniang panlungsod or
jurisdiction to be properly identified by metes and bounds or by more or less sangguniang bayan concerned.
permanent natural boundaries. Precisely because territorial jurisdiction is an
• (b) Boundary disputes involving two (2) or more municipalities within the same province
issue raised in the pending civil case, until and unless such issue is resolved shall be referred for settlement to the sangguniang panlalawigan concerned.
with finality, to define the territorial jurisdiction of the
proposed barangays would only be an exercise in futility. Not only that, we • (c) Boundary disputes involving municipalities or component cities of different provinces
would be paving the way for potentially ultra vires acts of shall be jointly referred for settlement to the sanggunians of the province concerned.
such barangays. Moreover, considering the expenses entailed in the holding • (d) Boundary disputes involving a component city or municipality on the one hand and a
of plebiscites, it is far more prudent to hold in abeyance the conduct of the highly urbanized city on the other, or two (2) or more highly urbanized cities, shall be
same, pending final determination of whether or not the entire area of the jointly referred for settlement to the respective sanggunians of the parties.
proposed barangays are truly within the territorial jurisdiction of the City of
Pasig. • (e) In the event the sanggunian fails to effect an amicable settlement within sixty (60)
days from the date the dispute was referred thereto, it shall issue a certification to that
effect. Thereafter, the dispute shall be formally tried by the sanggunian concerned which
2. Pateros v CA shall decide the issue within sixty (60) days from the date of the certification referred to
above.
a) The jurisdiction of the RTC was clarified in Municipality of Kananga v. Judge
Madrona, where this Court held that, even in the absence of any specific • Section 119. Appeal. - Within the time and manner prescribed by the Rules of Court, any
provision of law, RTCs have general jurisdiction to adjudicate all controversies party may elevate the decision of the sanggunian concerned to the proper Regional Trial
except those expressly withheld from their plenary powers. They have the Court having jurisdiction over the area in dispute. The Regional Trial Court shall decide the
power not only to take judicial cognizance of a case instituted for judicial appeal within one (1) year from the filing thereof. Pending final resolution of the disputed
action for the first time, but also to do so to the exclusion of all other courts at area prior to the dispute shall be maintained and continued for all legal purposes.
that stage. Indeed, the power is not only original, but also exclusive.
I. Succession of elective officials
3. Nueva Era v Municipality of Marcos 1. Navarro v CA
a) the CA need not treat the appeal via petition for review filed by Marcos as a a) Under Section 44, a permanent vacancy arises when an elective official fills a
petition for certiorari to be able to pass upon the same. B.P. Blg. 129, as higher vacant office, refuses to assume office, fails to qualify, dies, is removed
amended, which is supplemented by Rule 42 of the Rules of Civil Procedure, from office, voluntarily resigns, or is otherwise permanently incapacitated to
gives the CA the authority to entertain appeals of such judgments and final discharge the functions of his office.
orders rendered by the RTC in the exercise of its appellate jurisdiction. b) What is crucial is the interpretation of Section 45 (b) providing that "xxx only
the nominee of the political party under which the Sanggunian member
concerned has been elected and whose elevation to the position next higher
• Section 118. Jurisdictional Responsibility for Settlement of Boundary Dispute. - Boundary
in rank created the last vacancy in the Sanggunian shall be appointed in the
disputes between and among local government units shall, as much as possible, be
manner hereinabove provided. The appointee shall come from the political
settled amicably. To this end:
party as that of the Sanggunian member who caused the vacancy xxx.”

Page 25 of 48
c) The reason behind the right given to a political party to nominate a 4. Gamboa v Aguirre
replacement where a permanent vacancy occurs in the Sanggunian is to a) A vice-governor who is an acting governor cannot act as presiding officer of
maintain the party representation as willed by the people in the election. sangguinian. Rules on separation of powers apply.

2. Farinas v Arba
• Section 44. Permanent Vacancies in the Offices of the Governor, Vice-Governor, Mayor,
a) I. Where the Permanent Vacancy is Caused by a Sanggunian Member and Vice-Mayor. -
belonging to a Political Party
• If a permanent vacancy occurs in the office of the governor or mayor, the vice-governor
(1) Sangguniang Panlalawigan and Sangguniang Panlungsod of highly or vice-mayor concerned shall become the governor or mayor. If a permanent vacancy
urbanized cities and independent component cities - The President, occurs in the offices of the governor, vice-governor, mayor, or vice-mayor, the highest
through the Executive Secretary, upon the nomination and certification of ranking sanggunian member or, in case of his permanent inability, the second highest
the political party to which the member who caused the vacancy belonged, ranking sanggunian member, shall become the governor, vice-governor, mayor or vice-
as provided in 45(b). mayor, as the case may be. Subsequent vacancies in the said office shall be filled
(2) Sangguniang Panlungsod of component cities and Sangguniang Bayan - automatically by the other sanggunian members according to their ranking as defined
The Governor upon the nomination and certification of the political party to herein.
which the member who caused the vacancy belonged, as provided in • (b) If a permanent vacancy occurs in the office of the punong barangay, the highest
Sec. 45(b). ranking sanggunian barangay member or, in case of his permanent inability, the second
b) Where the Vacancy is Caused by a Sanggunian Member Not Belonging to a highest ranking sanggunian member, shall become the punong barangay.
Political Party • (c) A tie between or among the highest ranking sanggunian members shall be resolved
(1) A. Sangguniang Panlalawigan and Sangguniang Panlungsod of highly by the drawing of lots.
urbanized and independent component cities - The President, through the • (d) The successors as defined herein shall serve only the unexpired terms of their
Executive Secretary, upon recommendation of the Sangguniang predecessors.
Panlalawigan or Sangguniang Panlungsod as the case may be.
• For purposes of this Chapter, a permanent vacancy arises when an elective local official
(2) B. Sangguniang Panlungsod of component cities and Sangguniang Bayan fills a higher vacant office, refuses to assume office, fails to qualify, dies, is removed
- The Governor upon recommendation of the Sangguniang Panlungsod or from office, voluntarily resigns, or is otherwise permanently incapacitated to discharge
Sangguniang Bayan as the case may be. the functions of his office.
c) III. Where the Vacancy is Caused by a Member of the Sangguniang Barangay • For purposes of succession as provided in the Chapter, ranking in the sanggunian shall
- City or Municipal Mayor upon recommendation of the Sangguniang be determined on the basis of the proportion of votes obtained by each winning
Barangay. candidate to the total number of registered voters in each district in the immediately
preceding local election.

3. Damasen v Tumamao • Section 45. Permanent Vacancies in the Sanggunian. -


a) 2 Requirements: • (a) Permanent vacancies in the sanggunian where automatic succession provided
above do not apply shall be filled by appointment in the following manner:
(1) Nomination
(2) Certificate of membership from highest official of political party
Page 26 of 48
• (1) The President, through the Executive Secretary, in the case of the sangguniang the local chief executive concerned shall also submit necessary documents showing
panlalawigan and the sangguniang panlungsod of highly urbanized cities and that said legal causes no longer exist.
independent component cities;
• (c) When the incumbent local chief executive is traveling within the country but outside
• (2) The governor, in the case of the sangguniang panlungsod of component cities and his territorial jurisdiction for a period not exceeding three (3) consecutive days, he may
the sangguniang bayan; designate in writing the officer-in-charge of the said office. Such authorization shall
specify the powers and functions that the local official concerned shall exercise in the
• (3) The city or municipal mayor, in the case of sangguniang barangay, upon
recommendation of the sangguniang barangay concerned. absence of the local chief executive except the power to appoint, suspend, or dismiss
employees.
• (b) Except for the sangguniang barangay, only the nominee of the political party under
which the sanggunian member concerned had been elected and whose elevation to the • (d) In the event, however, that the local chief executive concerned fails or refuses to
position next higher in rank created the last vacancy in the sanggunian shall be issue such authorization, the vice-governor, the city or municipal vice-mayor, or the
appointed in the manner hereinabove provided. The appointee shall come from the highest ranking sangguniang barangay member, as the case may be, shall have the
same political party as that of the sanggunian member who caused the vacancy and right to assume the powers, duties, and functions of the said office on the fourth (4th)
shall serve the unexpired term of the vacant office. In the appointment herein day of absence of the said local chief executive, subject to the limitations provided in
mentioned, a nomination and a certificate of membership of the appointee from the subsection (c) hereof.
highest official of the political party concerned are conditions sine qua non, and any • (e) Except as provided above, the local chief executive shall in no case authorize any
appointment without such nomination and certification shall be null and void ab initio and local official to assume the powers, duties, and functions of the office, other than the
shall be a ground for administrative action against the official responsible therefore. vice-governor, the city or municipal vice-mayor, or the highest ranking sangguniang
barangay member, as the case may be.
• (c) In case or permanent vacancy is caused by a sanggunian member who does not
belong to any political party, the local chief executive shall, upon recommendation of the • Section 47. Approval of Leaves of Absence. -
sanggunian concerned, appoint a qualified person to fill the vacancy.
• (a) Leaves of absence of local elective officials shall be approved as follows:
• (d) In case of vacancy in the representation of the youth and the barangay in the • (1) Leaves of absence of the governor and the mayor of a highly urbanized city or an
sanggunian, said vacancy shall be filled automatically by the official next in rank of the independent component city shall be approved by the President or his duly authorized
organization concerned. representative;
• Section 46. Temporary Vacancy in the Office of the Local Chief Executive. - • (2) Leaves of absence of vice-governor or a city or municipal vice-mayor shall be
• (a) When the governor, city or municipal mayor, or punong barangay is temporarily approved by the local chief executive concerned: Provided, That the leaves of
incapacitated to perform his duties for physical or legal reasons such as, but not limited absence of the members of the sanggunian and its employees shall be approved by
to, leave of absence, travel abroad, and suspension from office, the vice-governor, city the vice-governor or city or municipal vice-mayor concerned;
or municipal vice-mayor, or the highest ranking sangguniang barangay member shall
• (3) Leaves of absence of the component city or municipal mayor shall be approved by
automatically exercise the powers and perform the duties and functions of the local chief the governor; and
executive concerned, except the power to appoint, suspend, or dismiss employees
which can only be exercised if the period of temporary incapacity exceeds thirty (30) • (4) Leaves of absence of a punong barangay shall be approved by the city or
working days. municipal mayor: Provided, That leaves of absence of sangguniang barangay
members shall be approved by the punong barangay.
• (b) Said temporary incapacity shall terminate upon submission to the appropriate
sanggunian of a written declaration by the local chief executive concerned that he has • (b) Whenever the application for leave of absence hereinabove specified is not acted
reported back to office. In cases where the temporary incapacity is due to legal causes, upon within five (5) working days after receipt thereof, the application for leave of
absence shall be deemed approved.
Page 27 of 48
c) A domicile of origin is acquired by every person at birth. It is usually the place
J. Qualifications and Disqualifications of local officials where the childs parents reside and continues until the same is abandoned by
acquisition of new domicile (domicile of choice). In Coquilla, the Court already
1. Jalosjos v COMELEC, 2012 acknowledged that for an individual to acquire American citizenship, he must
a) There is no hard and fast rule to determine a candidates compliance with establish residence in theUSA. Since Ty himself admitted that he became a
residency requirement since the question of residence is a question of naturalized American citizen, then he must have necessarily abandoned
intention.[12] Still, jurisprudence has laid down the following guidelines: (a) the Municipality of General Macarthur, Eastern Samar, Philippines, as his
every person has a domicile or residence somewhere; (b) where once domicile of origin; and transferred to the USA, as his domicile of choice.
established, that domicile remains until he acquires a new one; and (c) a
person can have but one domicile at a time.
4. Aratea v COMELEC
a) Section 65 of the Omnibus Election Code points to the Local Government
2. Sabili v COMELEC Code for the qualifications of elective local officials. A petition for
a) To establish a new domicile of choice, personal presence in the place must disqualification under Section 68 clearly refers to “the commission of
be coupled with conduct indicative of the intention to make it one's fixed and prohibited acts and possession of a permanent resident status in a foreign
permanent place of abode.[53] As in all administrative cases, the quantum of country.”20 All the offenses mentioned in Section 68 refer to election offenses
proof necessary in election cases is substantial evidence, or such relevant under the Omnibus Election Code, not to violations of other penal laws. There
evidence as a reasonable mind will accept as adequate to support a is absolutely nothing in the language of Section 68 that would justify including
conclusion.[54] violation of the three-term limit rule, or conviction by final judgment of the
crime of falsification under the Revised Penal Code, as one of the grounds or
offenses covered under Section 68.
3. Lapzon v COMELEC b) Clearly, the violation by Lonzanida of the three-term limit rule, or his conviction
a) Republic Act No. 9225 imposes no residency requirement for the reacquisition by final judgment of the crime of falsification under the Revised Penal Code,
or retention of Philippine citizenship; nor does it mention any effect of such does not constitute a ground for a petition under Section 68.
reacquisition or retention of Philippine citizenship on the current residence of
the concerned natural-born Filipino. Clearly, Republic Act No. 9225 treats
citizenship independently of residence. This is only logical and consistent with 5. Rodriguez v COMELEC
the general intent of the law to allow for dual citizenship. Since a natural-born a) Fugitive from justice includes not only those who flee after conviction to avoid
Filipino may hold, at the same time, both Philippine and foreign citizenships, punishment but likewise who, after being charged, flee to avoid prosecution.
he may establish residence either in the Philippines or in the foreign country of
b)
which he is also a citizen.
6. Mercado v Manzano
b) The term residence is to be understood not in its common acceptation as
referring to dwelling or habitation, but rather to domicile or legal residence, a) To begin with, dual citizenship is different from dual allegiance. The former
that is, the place where a party actually or constructively has his permanent arises when, as a result of the concurrent application of the different laws of
home, where he, no matter where he may be found at any given time, two or more states, a person is simultaneously considered a national by
eventually intends to return and remain (animus manendi) the said states. For instance, such a situation may arise when a person
whose parents are citizens of a state which adheres to the principle of jus
sanguinis is born in a state which follows the doctrine of jus soli. Such a
Page 28 of 48
person, ipso facto and without any voluntary act on his part, is concurrently persuade. Although immigration to the USA through a petition filed by a
considered a citizen of both states.  family member (sponsor) is allowed by USA immigration laws,[16] the
b) Dual allegiance, on the other hand, refers to the situation in which a person petitioned party is very much free to accept or reject the grant of resident
simultaneously owes, by some positive act, loyalty to two or more status. Permanent residency in the USA is not conferred upon the
states. While dual citizenship is involuntary, dual allegiance is the result of an unwilling; unlike citizenship, it is not bestowed by operation of law.[17] And
individuals volition. to reiterate, a person can have only one residence or domicile at any given
time.
c) the phrase dual citizenship in R.A. No. 7160, 40(d) and in R.A. No. 7854,
20 must be understood as referring to dual allegiance. Consequently,
persons with mere dual citizenship do not fall under this 8. Jalosjos v COMELEC, 2013
disqualification. Unlike those with dual allegiance, who must, therefore, be a) In particular, while Section 40(a) of the LGC allows a prior convict to run for
subject to strict process with respect to the termination of their status, for local elective office after the lapse of two (2) years from the time he serves his
candidates with dual citizenship, it should suffice if, upon the filing of their sentence, the said provision should not be deemed to cover cases wherein
certificates of candidacy, they elect Philippine citizenship to terminate their the law26 imposes a penalty, either as principal or accessory,27 which has the
status as persons with dual citizenship considering that their condition is the effect of disqualifying the convict to run for elective office. An example of this
unavoidable consequence of conflicting laws of different states. As Joaquin G. would be Article 41 of the RPC, which imposes the penalty of
Bernas, one of the most perceptive members of the Constitutional perpetual28 absolute29 disqualification as an accessory to the principal
Commission, pointed out: [D]ual citizenship is just a reality imposed on us penalties of reclusion perpetua and reclusion temporal:
because we have no control of the laws on citizenship of other countries. We
recognize a child of a Filipino mother. But whether or not she is considered a
citizen of another country is something completely beyond our control. 9. People v Sandiganbayan
a) On the contrary, legal disqualification in Article 244 of the Revised Penal
7. Ugdoracion v COMELEC Code simply means disqualification under the law. Clearly, Section 6, Article IX
of the 1987 Constitution and Section 94(b) of the Local Government Code of
a) Rules on domicile 1991 prohibits losing candidates within one year after such election to be
(1) Domicile is classified into (1) domicile of origin, which is acquired by every appointed to any office in the government or any government-owned or
person at birth; (2) domicile of choice, which is acquired upon controlled corporations or in any of their subsidiaries.
abandonment of the domicile of origin; and (3) domicile by operation of
law, which the law attributes to a person independently of his residence or
intention.
(2) In a controversy such as the one at bench, given the parties naturally • Section 39. Qualifications. -
conflicting perspectives on domicile, we are guided by three basic rules, • (a) An elective local official must be a citizen of the Philippines; a registered voter in the
namely: (1) a man must have a residence or domicile somewhere; (2) barangay, municipality, city, or province or, in the case of a member of the sangguniang
domicile, once established, remains until a new one is validly acquired; panlalawigan, sangguniang panlungsod, or sangguniang bayan, the district where he
and (3) a man can have but one residence or domicile at any given time intends to be elected; a resident therein for at least one (1) year immediately preceding
(3) The contention that Ugdoracions USA resident status was acquired the day of the election; and able to read and write Filipino or any other local language or
involuntarily, as it was simply the result of his sisters beneficence, does not dialect.

Page 29 of 48
• (b) Candidates for the position of governor, vice-governor, or member of the • (b) The regular members of the sangguniang panlalawigan, sangguniang panlungsod,
sangguniang panlalawigan, or mayor, vice-mayor or member of the sangguniang and sangguniang bayan shall be elected by district, as may be provided for by law.
panlungsod of highly urbanized cities must be at least twenty-one (21) years of age on Sangguniang barangay members shall be elected at large. The presidents of the
election day. leagues of sanggunian members of component cities and municipalities shall serve as
ex officio members of the sangguniang panlalawigan concerned. The presidents of the
• (c) Candidates for the position of mayor or vice-mayor of independent component cities,
component cities, or municipalities must be at least twenty-one (21) years of age on "liga ng mga barangay and the pederasyon ng mga sangguniang kabataan" elected by
election day. their respective chapters, as provided in this Code, shall serve as ex officio members of
the sangguniang panlalawigan, sangguniang panlungsod, and sangguniang bayan.
• (d) Candidates for the position of member of the sangguniang panlungsod or
sangguniang bayan must be at least eighteen (18) years of age on election day. • (c) In addition thereto, there shall be one (1) sectoral representative from the women,
one (1) from the workers, and one (1) from any of the following sectors: the urban poor,
• (e) Candidates for the position of punong barangay or member of the sangguniang indigenous cultural communities, disabled persons, or any other sector as may be
barangay must be at least eighteen (18) years of age on election day. determined by the sanggunian concerned within ninety (90) days prior to the holding of
• (f) Candidates for the sangguniang kabataan must be at least fifteen (15) years of age the next local elections as may be provided for by law. The COMELEC shall promulgate
but not more than twenty-one (21) years of age on election day. the rules and regulations to effectively provide for the election of such sectoral
representatives.
• Section 40. Disqualifications. - The following persons are disqualified from running for any
elective local position: • Section 42. Date of Election. - Unless otherwise provided by law, the elections for local
officials shall be held every three (3) years on the second Monday of May.
• (a) Those sentenced by final judgment for an offense involving moral turpitude or for an
offense punishable by one (1) year or more of imprisonment, within two (2) years after • Section 43. Term of Office. -
serving sentence; • (a) The term of office of all local elective officials elected after the effectivity of this Code
• (b) Those removed from office as a result of an administrative case; shall be three (3) years, starting from noon of June 30, 1992 or such date as may be
provided for by law, except that of elective barangay officials: Provided, That all local
• (c) Those convicted by final judgment for violating the oath of allegiance to the Republic;
officials first elected during the local elections immediately following the ratification of the
• (d) Those with dual citizenship; 1987 Constitution shall serve until noon of June 30, 1992.
• (e) Fugitives from justice in criminal or non-political cases here or abroad; • (b) No local elective official shall serve for more than three (3) consecutive terms in the
• (f) Permanent residents in a foreign country or those who have acquired the right to same position. Voluntary renunciation of the office for any length of time shall not be
reside abroad and continue to avail of the same right after the effectivity of this Code; considered as an interruption in the continuity of service for the full term for which the
and elective official concerned was elected.

• (g) The insane or feeble-minded. • (c) The term of office of barangay officials and members of the sangguniang kabataan
shall be for three (3) years, which shall begin after the regular election of barangay
• Section 41. Manner of Election. - officials on the second Monday of May 1994.
• (a) The governor, vice-governor, city mayor, city vice-mayor, municipal mayor, municipal • Section 76. Organizational Structure and Staffing Pattern. - Every local government unit
vice-mayor, and punong barangay shall be elected at large in their respective units by shall design and implement its own organizational structure and staffing pattern taking into
the qualified voters therein. However, the sangguniang kabataan chairman for each consideration its service requirements and financial capability, subject to the minimum
barangay shall be elected by the registered voters of the katipunan ng kabataan, as standards and guidelines prescribed by the Civil Service Commission.
provided in this Code.

Page 30 of 48
• Section 77. Responsibility for Human Resources and Development. - The chief executive of office of those approving such increase shall have expired: Provided, further, That the
of every local government unit shall be responsible for human resources and development increase in compensation of the appointive officials and employees shall take effect as
in his unit and shall take all personnel actions in accordance with the Constitutional provided in the ordinance authorizing such increase: Provided, however, That said
provisions on civil service, pertinent laws, and rules and regulations thereon, including increases shall not exceed the limitations on budgetary allocations for personal services
such policies, guidelines and standards as the Civil Service Commission may establish: provided under Title Five, Book II of this Code: Provided, finally, That such compensation
Provided, That the local chief executive may employ emergency or casual employees or may be based upon the pertinent provisions of Republic Act Numbered Sixty-seven fifty-
laborers paid on a daily wage or piecework basis and hired through job orders for local eight (R.A. No 6758), otherwise known as the "Compensation and Position Classification
projects authorized by the sanggunian concerned, without need of approval or attestation Act of 1989”.

by the Civil Service Commission: Provided, further, That the period of employment of The punong barangay, the sangguniang barangay member, the sangguniang kabataan
emergency or casual laborers as provided in this Section shall not exceed six (6) months.
 chairman, the barangay treasurer, and the barangay secretary shall be entitled to such
The Joint Commission on Local Government Personnel Administration organized pursuant compensation, allowances, emoluments, and such other privileges as provided under Title
to Presidential Decree Numbered Eleven Hundred thirty-six (P.D. No. 1136) is hereby One Book III of this Code.

abolished and its personnel, records, equipment and other assets transferred to the Elective local officials shall be entitled to the same leave privileges as those enjoyed by
appropriate office in the Civil Service Commission. appointive local officials, including the cumulation and commutation thereof.
• Section 78. Civil Service Law, Rules and Regulations, and Other Related Issuances. - All • Section 82. Resignation of Elective Local Officials. -
matters pertinent to human resources and development in local government units shall be
• (a) Resignations by elective local officials shall be deemed effective only upon
governed by the civil service law and such rules and regulations and other issuances acceptance by the following authorities:
promulgated pursuant thereto, unless otherwise specified in this Code.
• (1) The President, in the case of governors, vice-governors, and mayors and vice-
• Section 79. Limitation to Appointments. - No person shall be appointed in the career mayors of highly urbanized cities and independent component cities;
service of the local government if he is related within the fourth civil degree of
consanguinity or affinity to the appointing or recommending authority. • (2) The governor, in the case of municipal mayors, municipal vice-mayors, city
mayors and city vice-mayors of component cities;
• Section 80. Public Notice of Vacancy; Personnel Selection Board. -
• (3) The sanggunian concerned, in the case of sanggunian members; and
• (a) Whenever a local executive decides to fill a vacant career position, there shall be
posted notices of the vacancy in at least three (3) conspicuous public places in the local • (4) The city or municipal mayor, in the case of barangay officials.
government unit concerned for a period of not less than fifteen (15) days. • (b) Copies of the resignation letters of elective local officials, together with the action
• (b) There shall be established in every province, city or municipality a personnel taken by the aforesaid authorities, shall be furnished the Department of the Interior and
selection board to assist the local chief executive in the judicious and objective selection Local Government.
or personnel for employment as well as for promotion, and in the formulation of such • (c) The resignation shall be deemed accepted if not acted upon by the authority
policies as would contribute to employee welfare. concerned within fifteen (15) days from receipt thereof.
• (c) The personnel selection board shall be headed by the local chief executive, and its • (d) Irrevocable resignations by sanggunian members shall be deemed accepted upon
members shall be determined by resolution of the sanggunian concerned. A presentation before an open session of the sanggunian concerned and duly entered in
representative of the Civil Service Commission, if any, and the personnel officer of the its records: Provided, however, That this subsection does not apply to sanggunian
local government unit concerned shall be ex officio members of the board. members who are subject to recall elections or to cases where existing laws prescribed
• Section 81. Compensation of Local Officials and Employees. - The compensation of local the manner of acting upon such resignations.
officials and personnel shall be determined by the sanggunian concerned: Provided, That
the increase in compensation of elective local officials shall take effect only after the terms
Page 31 of 48
• Section 83. Grievance Procedure. - In every local government unit, the local chief • Section 88. Execution Pending Appeal. - An appeal shall not prevent the execution of a
executive shall establish a procedure to inquire into, act upon, resolve or settle complaints decision of removal or suspension of a respondent-appellant. In case the respondent-
and grievances presented by local government employees. appellant is exonerated, he shall be reinstated to his position with all the rights and
privileges appurtenant thereto from the time he had been deprived thereof.
• Section 84. Administrative Discipline. - Investigation and adjudication of administrative
complaints against appointive local officials and employees as well as their suspension • Section 89. Prohibited Business and Pecuniary Interest. -
and removal shall be in accordance with the civil service law and rules and other pertinent
• (a) It shall be unlawful for any local government official or employee, directly or
laws. The results of such administrative investigations shall be reported to the Civil Service indirectly, to:
Commission.
• (1) Engage in any business transaction with the local government unit in which he is
• Section 85. Preventive Suspension of Appointive Local Officials and Employees. an official or employee or over which he has the power of supervision, or with any of
• (a) The local chief executives may preventively suspend for a period not exceeding sixty its authorized boards, officials, agents, or attorneys, whereby money is to be paid, or
(60) days and subordinate official or employee under his authority pending investigation property or any other thing of value is to be transferred, directly or indirectly, out of the
if the charge against such official or employee involves dishonesty, oppression or grave resources of the local government unit to such person or firm;
misconduct or neglect in the performance of duty, or if there is reason to believe that the
• (2) Hold such interests in any cockpit or other games licensed by a local government
respondent is guilty of the charges which would warrant his removal from the service. unit;
• (b) Upon expiration of the preventive suspension, the suspended official or employee • (3) Purchase any real estate or other property forfeited in favor of such local
shall be automatically reinstated in office without prejudice to the continuation of the government unit for unpaid taxes or assessment, or by virtue of a legal process at the
administrative proceedings against him until its termination. If the delay in the instance of the said local government unit;
proceedings of the case is due to the fault, neglect or request of the respondent, the
time of the delay shall not be counted in computing the period of suspension herein • (4) Be a surety for any person contracting or doing business with the local
provided. government unit for which a surety is required; and
• Section 86. Administrative Investigation. - In any local government unit, administrative • (5) Possess or use any public property of the local government unit for private
investigation may be conducted by a person or a committee duly authorized by the local purposes.
chief executive. Said person or committee shall conduct hearings on the cases brought • (b) All other prohibitions governing the conduct of national public officers relating to
against appointive local officials and employees and submit their findings and prohibited business and pecuniary interest so provided for under Republic Act
recommendations to the local chief executive concerned within fifteen (15) days from the Numbered Sixty-seven thirteen (R.A. No. 6713) otherwise known as the "Code of
conclusion of the hearings. The administrative cases herein mentioned shall be decided Conduct and Ethical Standards for Public Officials and Employees" and other laws shall
within ninety (90) days from the time the respondent is formally notified of the charges. also be applicable to local government officials and employees.
• Section 87. Disciplinary Jurisdiction. - Except as otherwise provided by law, the local chief • Section 90. Practice of Profession. -
executive may impose the penalty of removal from service, demotion in rank, suspension
for not more than one (1) year without pay, fine in an amount not exceeding six (6) months • (a) All governors, city and municipal mayors are prohibited from practicing their
profession or engaging in any occupation other than the exercise of their functions as
salary, or reprimand and otherwise discipline subordinate officials and employees under
local chief executives.
his jurisdiction. If the penalty imposed is suspension without pay for not more than thirty
(30) days, his decision shall be final. If the penalty imposed is heavier than suspension of • (b) Sanggunian members may practice their professions, engage in any occupation, or
thirty (30) days, the decision shall be appealable to the Civil Service Commission, which teach in schools except during session hours: Provided, That sanggunian members who
shall decide the appeal within thirty (30) days from receipt thereof. are also members of the Bar shall not:

Page 32 of 48
• (1) Appear as counsel before any court in any civil case wherein a local government Unless otherwise allowed by law or by the primary functions of his position, no elective or
unit or any office, agency, or instrumentality of the government is the adverse party; appointive local official shall hold any other office or employment in the government or any
subdivision, agency or instrumentality thereof, including government-owned or controlled
• (2) Appear as counsel in any criminal case wherein an officer or employee of the
national or local government is accused of an offense committed in relation to his corporations or their subsidiaries.
office. • Section 95. Additional or Double Compensation. - No elective or appointive local official or
employee shall receive additional, double, or indirect compensation, unless specifically
• (3) Collect any fee for their appearance in administrative proceedings involving the
local government unit of which he is an official; and authorized by law, nor accept without the consent of Congress, any present, emoluments,
office, or title of any kind from any foreign government. Pensions or gratuities shall not be
• (4) Use property and personnel of the government except when the sanggunian considered as additional, double, or indirect compensation.
member concerned is defending the interest of the government.
• Section 96. Permission to Leave Station. -
• (c) Doctors of medicine may practice their profession even during official hours of work
only on occasions of emergency: Provided, That the officials concerned do not derive • (a) Provincial, city, municipal, and barangay appointive officials going on official travel
monetary compensation therefrom. shall apply and secure written permission from their respective local chief executives
before departure. The application shall specify the reasons for such travel, and the
• Section 91. Statement of Assets and Liabilities. - (a) Officials and employees of local permission shall be given or withheld based on considerations of public interest,
government units shall file sworn statements of assets, liabilities and net worth, lists of financial capability of the local government unit concerned and urgency of the travel.
relatives within the fourth civil degree of consanguinity or affinity in government service,
financial and business interests, and personnel data sheets as required by law. • Should the local chief executive concerned fall to act upon such application within four
(4) working days from receipt thereof, it shall be deemed approved.
• Section 92. Oath of Office. - (a) All elective and appointive local officials and employees
shall, upon assumption to office, subscribe to an oath or affirmation of office in the • (b) Mayors of component cities and municipalities shall secure the permission of the
prescribed form. The oath or affirmation of office shall be filed with the office of the local governor concerned for any travel outside the province.
chief executive concerned. A copy of the oath or affirmation of office of all elective and • (c) Local government officials traveling abroad shall notify their respective sanggunian:
appointive local officials and employees shall be preserved in the individual personal Provided, That when the period of travel extends to more than three (3) months, during
records file under the custody of the personnel office, division, or section of the local periods of emergency or crisis or when the travel involves the use of public funds,
government unit concerned. permission from the Office of the President shall be secured.
• Section 93. Partisan Political Activity. - No local official or employee in the career civil • (d) Field officers of national agencies or offices assigned in provinces, cities, and
service shall engage directly or indirectly in any partisan political activity or take part in any municipalities shall not leave their official stations without giving prior written notice to
election, initiative, referendum, plebiscite, or recall, except to vote, nor shall he use his the local chief executive concerned. Such notice shall state the duration of travel and
official authority or influence to cause the performance of any political activity by any the name of the officer whom he shall designate to act for and in his behalf during his
person or body. He may, however, express his views on current issues, or mention the absence.
names of certain candidates for public office whom he supports. Elective local officials may
• Section 97. Annual Report. - On or before March 31 of each year, every local chief
take part in partisan political and electoral activities, but it shall be unlawful for them to executive shall submit an annual report to the sanggunian concerned on the socio-
solicit contributions from their subordinates or subject these subordinates to any of the economic, political and peace and order conditions, and other matters concerning the local
prohibited acts under the Omnibus Election Code. government unit, which shall cover the immediately preceding calendar year. A copy of the
• Section 94. Appointment of Elective and Appointive Local Officials; Candidates Who Lost report shall be forwarded to the Department of the Interior and Local Government.
in an Election. - (a) No elective or appointive local official shall be eligible for appointment Component cities and municipalities shall likewise provide the sangguniang panlalawigan
or designation in any capacity to any public office or position during his tenure.
 copies of their respective annual reports.

Page 33 of 48
• Section 387. Chief Officials and Offices. - • (b) The barangay secretary shall be of legal age, a qualified voter and an actual resident
of the barangay concerned.
• (a) There shall be in each barangay a punong barangay, seven (7) sangguniang
barangay members, the sangguniang kabataan chairman, a barangay secretary, and a • (c) No person shall be appointed barangay secretary if he is a sangguniang barangay
barangay treasurer. member, a government employee, or a relative of the punong barangay within the fourth
civil degree of consanguinity of affinity.
• (b) There shall also be in every barangay a lupong tagapamayapa. The sangguniang
barangay may form community brigades and create such other positions or offices as • (d) The barangay secretary shall:
may be deemed necessary to carry out the purposes of the barangay government in
• Section 395. Barangay Treasurer: Appointment, Qualification, Powers and Duties. -
accordance with the needs of public service, subject to the budgetary limitations on
personal services prescribed under Title Five, Book II of this Code. • (a) The barangay treasurer shall be appointed by the punong barangay with the
concurrence of the majority of all the sangguniang barangay members. The appointment
• Section 388. Persons in Authority. - For purposes of the Revised Penal Code, the punong of the barangay treasurer shall not be subject to attestation by the Civil Service
barangay, sangguniang barangay members, and members of the lupong tagapamayapa in Commission.
each barangay shall be deemed as persons in authority in their jurisdictions, while other
barangay officials and members who may be designated by law or ordinance and charged • (b) The barangay treasurer shall be of legal age, a qualified voter, and an actual resident
with the maintenance of public order, protection and security of life and property, or the of the barangay concerned.
maintenance of a desirable and balanced environment, and any barangay member who • (c) No person shall be appointed barangay treasurer if he is a sangguniang barangay
comes to the aid of persons in authority, shall be deemed agents of persons in authority. member, a government employee, or a relative of the punong barangay within the fourth
• Section 389. Chief Executive: Powers, Duties, and Functions. - civil degree of consanguinity or affinity.
• Section 390. Composition. - The sangguniang barangay, the legislative body of the • (d) The barangay treasurer shall be bonded in accordance with existing laws in an
barangay, shall be composed of the punong barangay as presiding officer, and the seven amount to be determined by the sangguniang barangay but not exceeding Ten thousand
(7) regular sangguniang barangay members elected at large and sangguniang kabataan pesos (P10,000.00), premiums for which shall be paid by the barangay.
chairman, as members. • (e) The barangay treasurer shall:
• Section 391. Powers, Duties, and Functions. - • Section 396. Other Appointive Officials. - The qualifications, duties, and functions of all
• Section 392. Other Duties of Sangguniang Barangay Members. - In addition to their duties other barangay officials appointed by the punong barangay shall be governed by the
as members of the sangguniang barangay, sangguniang barangay members may: provisions of this Code and other laws or by barangay ordinances.
• (a) Assist the punong barangay in the discharge of his duties and functions; • Section 397. Composition; Meetings. -
• (b) Act as peace officers in the maintenance of public order and safety; and • Section 398. Powers of the Barangay Assembly. - The barangay assembly shall:
• (c) Perform such other duties and functions as the punong barangay may delegate. • (a) Initiate legislative processes by recommending to the sangguniang barangay the
adoption of measures for the welfare of the barangay and the city or municipality
• Section 393. Benefits of Barangay Officials. - concerned;
• Section 394. Barangay Secretary: Appointment, Qualifications, Powers and Duties. - • (b) Decide on the adoption of initiative as a legal process whereby the registered voters
• (a) The barangay secretary shall be appointed by the punong barangay with the of the barangay may directly propose, enact, or amend any ordinance; and
concurrence of the majority of all the sangguniang barangay members. The
• (c) Hear and pass upon the semestral report of the sangguniang barangay concerning
appointment of the barangay secretary shall not be subject to attestation by the Civil its activities and finances.
Service Commission.
• Section 443. Officials of the Municipal Government. -
Page 34 of 48
• (a) There shall be in each municipality a municipal mayor, a municipal vice-mayor, exercise of the corporate powers of the municipality as provided for under Section 22 of
sangguniang bayan members, a secretary to the sangguniang bayan, a municipal this Code, and shall:
treasurer, a municipal assessor, a municipal accountant, a municipal budget officer, a
• Section 454. Officials of the City Government.
municipal planning and development coordinator, a municipal engineer/building official,
a municipal health officer and a municipal civil registrar. • (a) There shall be in each city a mayor, a vice-mayor, sangguniang panlungsod
members, a secretary to the sangguniang panlungsod, a city treasurer, a city assessor,
• (b) In addition thereto, the mayor may appoint a municipal administrator, a municipal a city accountant, a city budget officer, a city planning and development coordinator, a
legal officer, a municipal agriculturist, a municipal environment and natural resources city engineer, a city health officer, a city civil registrar, a city administrator, a city legal
officer, a municipal social welfare and development officer, a municipal architect, and a officer, a city veterinarian, a city social welfare and development officer, and a city
municipal information officer. general services officer.
• (c) The sangguniang bayan may: • (b) In addition thereto, the city mayor may appoint a city architect, a city information
• (1) Maintain existing offices not mentioned in subsections (a) and (b) hereof; officer, a city agriculturist, a city population officer, a city environment and natural
resources officer, and a city cooperatives officer.
• (2) Create such other offices as may be necessary to carry out the purposes of the
municipal government; or • The appointment of a city population officer shall be optional in the city: Provided,
however, That cities which have existing population offices shall continue to maintain
• (3) Consolidate the functions of any office with those of another in the interest of
efficiency and economy. such offices for a period of five (5) years from the date of the effectivity of this Code,
after which said offices shall become optional.
• (d) Unless otherwise provided herein, heads of departments and offices shall be
appointed by the municipal mayor with the concurrence of the majority of all the • (c) The sangguniang panlungsod may:
sangguniang bayan members, subject to civil service law, rules and regulations. The • (1) Maintain existing offices not mentioned in subsections (a) and (b) hereof;
sangguniang bayan shall act on the appointment within fifteen (15) days from the date of
• (2) Create such other offices as may be necessary to carry out the purposes of the
its submission; otherwise, the same shall be deemed confirmed. city government; or
• (e) Elective and appointive municipal officials shall receive such compensation, • (3) Consolidate the functions of any office with those of another in the interest of
allowances and other emoluments as may be determined by law or ordinance, subject efficiency and economy.
to the budgetary limitations on personal services as prescribed in Title Five, Book Two of
this Code: Provided, That no increase in compensation of the mayor, vice-mayor, and • (d) Unless otherwise provided herein, heads of departments and offices shall be
sangguniang bayan members shall take effect until after the expiration of the full term of appointed by the city mayor with the concurrence of the majority of all the sangguniang
all the elective local officials approving such increase. panlungsod members, subject to civil service law, rules and regulations. The
sangguniang panlungsod shall act on the appointment within fifteen (15) days from the
• Section 444. The Chief Executive: Powers, Duties, Functions and Compensation. date of its submission, otherwise the same shall be deemed confirmed.
• Section 445. Powers, Duties and Compensation. -
• (e) Elective and appointive city officials shall receive such compensation, allowances,
• (a) The vice-mayor shall: and other emoluments as may be determined by law or ordinance, subject to the
budgetary limitations on personal services prescribed under Title Five, Book II of this
• Section 447. Powers, Duties, Functions and Compensation. -
Code: Provided, That, no increase in compensation of the mayor, vice-mayor and
• (a) The sangguniang bayan, as the legislative body of the municipality, shall enact sangguniang panlungsod members shall take effect until after the expiration of the full
ordinances, approve resolutions and appropriate funds for the general welfare of the term of the said local officials approving such increase.
municipality and its inhabitants pursuant to Section 16 of this Code and in the proper
• Section 455. Chief Executive; Powers, Duties and Compensation.
Page 35 of 48
• Section 456. Powers, Duties and Compensation. • (1) Maintain existing offices not mentioned in subsections (a) and (b) hereof;
• (a) The city vice-mayor shall: • (2) Create such other offices as may be necessary to carry out the purposes of the
provincial government; or
• (1) Be the presiding officer of the sangguniang panlungsod and sign all warrants drawn on
the city treasury for all expenditures appropriated for the operation of the sangguniang • (3) Consolidate the functions of any office with those of another in the interest of
panlungsod; efficiency and economy;
• Section 457. Composition • (d) Unless otherwise provided herein, heads of departments and offices shall be
appointed by the governor with the concurrence of the majority of all the sangguniang
• (a) The sangguniang panlungsod, the legislative body of the city, shall be composed of
the city vice-mayor as presiding officer, the regular sanggunian members, the president panlalawigan members, subject to civil service law, rules and regulations. The
of the city chapter of the liga ng mga barangay, the president of the panlungsod na sangguniang panlalawigan shall act on the appointment within fifteen (15) days from the
pederasyon ng mga sangguniang kabataan, and the sectoral representatives, as date of its submission; otherwise the same shall be deemed confirmed;
members. • (e) Elective and appointive provincial officials shall receive such compensation,
allowances, and other emoluments as may be determined by law or ordinance, subject
• (b) In addition thereto, there shall be three (3) sectoral representatives: one (1) from the
women; and as shall be determined by the sanggunian concerned within ninety (90) to the budgetary limitations on personal services prescribed under Title Five, Book II of
days prior to the holding of the local elections, one (1) from agricultural or industrial this Code: Provided, That, no increase in compensation shall take effect until after the
workers; and one (1) from the other sectors, including the urban poor, indigenous expiration of the full term of all the elective officials approving such increase.
cultural communities, or disabled persons. • Section 464. Residence and Office. - During the incumbency of the governor, he shall
have his official residence in the capital of the province. All elective and appointive
• (c) The regular members of the sangguniang panlungsod and the sectoral
representatives shall be elected in the manner as may be provided for by law. provincial officials shall hold office in the provincial capital: Provided, That, upon resolution
of the sangguniang panlalawigan, elective and appointive provincial officials may hold
• Section 463. Officials of the Provincial Government. office in any component city or municipality within the province for a period of not more
• (a) There shall be in each province a governor, a vice-governor, members of the than seven (7) days for any given month.
sangguniang panlalawigan, a secretary to the sangguniang panlalawigan, a provincial • Section 465. The Chief Executive: Powers, Duties, Functions, and Compensation.
treasurer, a provincial assessor, a provincial accountant, a provincial engineer, a
provincial budget officer, a provincial planning and development coordinator, a provincial • Section 466. Powers, Duties, and Compensation.
legal officer, a provincial administrator, a provincial health officer, a provincial social • (a) The vice-governor shall:
welfare and development officer, a provincial general services officer, a provincial
• (1) Be the presiding officer of the sangguniang panlalawigan and sign all warrants
agriculturist, and a provincial veterinarian. drawn on the provincial treasury for all expenditures appropriated for the operation of
• (b) In addition thereto, the governor may appoint a provincial population officer, a the sangguniang panlalawigan;
provincial natural resources and environment officer, a provincial cooperative officer, a
• Section 467. Composition.
provincial architect, and a provincial information officer.
• (a) The sangguniang panlalawigan, the legislative body of the province, shall be
• The appointment of a provincial population officer shall be optional in the province: composed of the provincial vice-governor as presiding officer, the regular sanggunian
Provided, however, That provinces which have existing population offices shall continue members, the president of the provincial chapter of the liga ng mga barangay, the
to maintain such offices for a period of five (5) years from the date of the effectivity of president of the panlalawigang pederasyon ng mga sangguniang kabataan, the
this Code, after which said offices shall become optional. president of the provincial federation of sanggunian members of municipalities and
• (c) The sangguniang panlalawigan may: component cities and the sectoral representatives, as members.

Page 36 of 48
• (b) In addition thereto, there shall be three (3) sectoral representatives: one (1) from the a) Ganzon v CA
women; and as shall be determined by the sanggunian concerned within ninety (90) (1) The OP, through the DILG has the power to impose preventive
days prior to the holding of the local elections, one (1) from the agricultural or industrial suspension/discipline public official. That does not constitute control which
workers; and one (1) from other sectors including the urban poor, indigenous cultural is prohibited by the Constitution. OP merely exercises investigative powers
communities, or disabled persons. which is not necessarily included in exercising of control.
• (c) The regular members of the sangguniang panlalawigan and the sectoral (2) However, the OP may not impose 600 day suspension because it will
representatives shall be elected in the manner as may be provided for by law. amount to removal of the public official which the OP cannot do.
• Section 469. Qualifications, Powers and Duties.
• (a) There shall be a secretary to the sanggunian who shall be a career official with the b) Joson v Torres
rank and salary equal to a head of department or office.
(1) The power of the DILG to discipline local officials comes from the alter
• (b) No person shall be appointed secretary to the sanggunian unless he is a citizen of ego doctrine. Procedure is for elective officials only. The rules on
the Philippines, a resident of the local government unit concerned, of good moral appointive officials are more lax than those of elective officials.
character, a holder of a college degree preferably in law, commerce or public
administration from a recognized college or university, and a first grade civil service
eligible or its equivalent. c) Pablico v Villapando
• The appointment of a secretary to the sanggunian is mandatory for provincial, city and (1) The OP cannot remove an elective local official. Removal is proper only
municipal governments. through courts.
• (c) The secretary to the sanggunian shall take charge of the office of the secretary to the
sanggunian and shall:
d) Miranda v Sandiganbayan
(1) The Ombudsman is not governed by LGC therefore the limit on preventive
K. Discipline of local officials suspension under LGC does not apply to Ombudsman. Ombudsman law
1. Elective Officials applies. Ombudsman law may suspend for a maximum 6 months.
a) Requisites
b) Ultra Vires Contracts • REQUISITES OF OREVENTIVE SUSPENSION UNDER LGC
• 1. there is reasonable ground to believe that the respondent has committed the act or
(1) Grounds acts complained of;
(2) Jurisdiction • 2. the evidence of culpability is strong;
c) Preventive Suspension • 3. the gravity of the offense so warrants;
• 4. the continuance in office of the respondent could influence the witnesses or pose a
d) Removal threat to the safety and integrity of the records and other evidence.
e) Administrative Appeal and Exection Pending Appeal
• REQUISITES UNDER OMBUDSMAN LAW
f) Doctrine of Condonation
• 1. the charge against the officer or employee should involve dishonestly, oppression or
2. Appointive officials grave misconduct or neglect in the performance of duty;
• 2. that the charges should warrant removal from the service; or
Page 37 of 48
• 3. The respondent's continued stay in office would prejudice the case filed against him. • (c) Dishonesty, oppression, misconduct in office, gross negligence, or dereliction of duty;
• (d) Commission of any offense involving moral turpitude or an offense punishable by at
e) Lapid v CA least prision mayor;
(1) Section 68. Execution Pending Appeal. An appeal shall not prevent a • (e) Abuse of authority;
decision from becoming final and executory. The respondent shall be
• (f) Unauthorized absence for fifteen (15) consecutive working days, except in the case
considered as having been placed under preventive suspension during the of members of the sangguniang panlalawigan, sangguniang panlungsod, sangguniang
pendency of an appeal in the event he wins such appeal. In the event the bayan, and sangguniang barangay;
appeal results in an exoneration, he shall be paid his salary and such
other emoluments during the pendency of the appeal. • (g) Application for, or acquisition of, foreign citizenship or residence or the status of an
immigrant of another country; and
(2) However, since in this case the applicable law is the Ombudsman law, the
judgment is immediately executory and not stayed by an appeal. • (h) Such other grounds as may be provided in this Code and other laws.
• An elective local official may be removed from office on the grounds enumerated above
by order of the proper court.
f) Aguinaldo v Santos
• Section 61. Form and Filing of Administrative Complaints. - A verified complaint against
(1) Criminal offenses cannot be condoned i.e. not proper subjects of the any erring local elective official shall be prepared as follows:
doctrine of condonation
• (a) A complaint against any elective official of a province, a highly urbanized city, an
independent component city or component city shall be filed before the Office of the
g) Salumbides v Ombudsman President;
(1) Condonation does not apply to appointive officials and criminal cases. • (b) A complaint against any elective official of a municipality shall be filed before the
sangguniang panlalawigan whose decision may be appealed to the Office of the
President; and
h) Salalima v Guingona
• (c) A complaint against any elective barangay official shall be filed before the
(1) When an elective official is elected again subsequent to his administrative sangguniang panlungsod or sangguniang bayan concerned whose decision shall be
offense, he is deemed forgiven. However, the Doctrine of Condonation final and executory.
does not apply to appointive officials because they are not elected by the
people and therefore the people cannot forgive them. • Section 62. Notice of hearing. -
• (a) Within seven (7) days after the administrative complaint is filed, the Office of the
President or the sanggunian concerned, as the case may be, shall require the
i) (BINAY CASE OMBUDSMAN) respondent to submit his verified answer within fifteen (15) days from receipt thereof,
and commence the investigation of the case within ten (10) days after receipt of such
answer of the respondent.
• Section 60. Grounds for Disciplinary Actions. - An elective local official may be disciplined,
suspended, or removed from office on any of the following grounds: • (b) When the respondent is an elective official of a province or highly urbanized city,
such hearing and investigation shall be conducted in the place where he renders or
• (a) Disloyalty to the Republic of the Philippines; holds office. For all other local elective officials, the venue shall be the place where the
• (b) Culpable violation of the Constitution; sanggunian concerned is located.

Page 38 of 48
• (c) However, no investigation shall be held within ninety (90) days immediately prior to • Section 65. Rights of Respondent. - The respondent shall be accorded full opportunity to
any local election, and no preventive suspension shall be imposed within the said appear and defend himself in person or by counsel, to confront and cross-examine the
period. If preventive suspension has been imposed prior to the 90-day period witnesses against him, and to require the attendance of witnesses and the production of
immediately preceding local election, it shall be deemed automatically lifted upon the documentary process of subpoena or subpoena duces tecum.
start of aforesaid period.
• Section 66. Form and Notice of Decision. -
• Section 63. Preventive Suspension. - • (a) The investigation of the case shall be terminated within ninety (90) days from the
• (a) Preventive suspension may be imposed: start thereof. Within thirty (30) days after the end of the investigation, the Office of the
President or the sanggunian concerned shall render a decision in writing stating clearly
• (1) By the President, if the respondent is an elective official of a province, a highly
urbanized or an independent component city; and distinctly the facts and the reasons for such decision. Copies of said decision shall
immediately be furnished the respondent and all interested parties.
• (2) By the governor, if the respondent is an elective official of a component city or
municipality; or • (b) The penalty of suspension shall not exceed the unexpired term of the respondent or
a period of six (6) months for every administrative offense, nor shall said penalty be a
• (3) By the mayor, if the respondent is an elective official of the barangay. bar to the candidacy of the respondent so suspended as long as he meets the
• (b) Preventive suspension may be imposed at any time after the issues are joined, when qualifications required for the office.
the evidence of guilt is strong, and given the gravity of the offense, there is great • (c) The penalty of removal from office as a result of an administrative investigation shall
probability that the continuance in office of the respondent could influence the witnesses be considered a bar to the candidacy of the respondent for any elective position.
or pose a threat to the safety and integrity of the records and other evidence: Provided,
That, any single preventive suspension of local elective officials shall not extend beyond • Section 67. Administrative Appeals. - Decisions in administrative cases may, within thirty
sixty (60) days: Provided, further, That in the event that several administrative cases are (30) days from receipt thereof, be appealed to the following:
filed against an elective official, he cannot be preventively suspended for more than • (a) The sangguniang panlalawigan, in the case of decisions of the sangguniang
ninety (90) days within a single year on the same ground or grounds existing and known panlungsod of component cities and the sangguniang bayan; and
at the time of the first suspension.
• (b) The Office of the President, in the case of decisions of the sangguniang
• (c) Upon expiration of the preventive suspension, the suspended elective official shall be panlalawigan and the sangguniang panlungsod of highly urbanized cities and
deemed reinstated in office without prejudice to the continuation of the proceedings independent component cities.
against him, which shall be terminated within one hundred twenty (120) days from the
• Decisions of the Office of the President shall be final and executory.
time he was formally notified of the case against him. However, if the delay in the
proceedings of the case is due to his fault, neglect, or request, other than the appeal • Section 68. Execution Pending Appeal. - An appeal shall not prevent a decision from
duly filed, the duration of such delay shall not be counted in computing the time of becoming final or executory. The respondent shall be considered as having been placed
termination of the case. under preventive suspension during the pendency of an appeal in the event he wins such
appeal. In the event the appeal results in an exoneration, he shall be paid his salary and
• (d) Any abuse of the exercise of the power of preventive suspension shall be penalized such other emoluments during the pendency of the appeal.
as abuse of authority.
• Section 64. Salary of Respondent Pending Suspension. - The respondent official
preventively suspended from office shall receive no salary or compensation during such L. Recall
suspension; but upon subsequent exoneration and reinstatement, he shall be paid full 1. Garcia v COMELEC
salary or compensation including such emoluments accruing during such suspension.
a) Previously there are two modes of a recall:

Page 39 of 48
(1) 1. Initiative • (2) City level. - All punong barangay and sanggunian barangay members in the city;
(2) 2. By representative—Removed/repealed already by RA 9244 • (3) Legislative District level. - In case where sangguniang panlalawigan members are
b) Only ground for recall is loss of confidence elected by district, all elective municipal officials in the district; and in cases where
sangguniang panlungsod members are elected by district, all elective barangay
officials in the district; and
2. Adormeo v COMELEC • (4) Municipal level. - All punong barangay and sangguniang barangay members in the
a) Recall election served as interruption of the 3 term limit rule. municipality.
• (c) A majority of all the preparatory recall assembly members may convene in session in
a public place and initiate a recall proceedings against any elective official in the local
3. Socrates v COMELEC
government unit concerned. Recall of provincial, city, or municipal officials shall be
a) Winner in a recall election cannot be charged of a term for purposes of the 3 validly initiated through a resolution adopted by a majority of all the members of the
term limit. preparatory recall assembly concerned during its session called for the purpose.
• (d) Recall of any elective provincial, city, municipal, or barangay official may also be
4. Mendoza v COMELEC validly initiated upon petition of at least twenty-five percent (25%) of the total number of
registered voters in the local government unit concerned during the election in which the
a) Served means fully served. local official sought to be recalled was elected.
(1) 2 requisites for the 3 term limit rule • (1) A written petition for recall duly signed before the election registrar or his
(a) Official has been elected 3 consecutive times representative, and in the presence of a representative of the petitioner and a
representative of the official sought to be recalled and, and in a public place in the
(b) He has fully served the period of each term
province, city, municipality, or barangay, as the case may be, shall be filed with the
COMELEC through its office in the local government unit concerned. The COMELEC
or its duly authorized representative shall cause the publication of the petition in a
public and conspicuous place for a period of not less than ten (10) days nor more
than twenty (20) days, for the purpose of verifying the authenticity and genuineness of
• Section 69. By Whom Exercised. - The power of recall for loss of confidence shall be the petition and the required percentage of voters.
exercised by the registered voters of a local government unit to which the local elective
• (2) Upon the lapse of the aforesaid period, the COMELEC or its duly authorized
official subject to such recall belongs. representative shall announce the acceptance of candidates to the position and
• Section 70. Initiation of the Recall Process. - thereafter prepare the list of candidates which shall include the name of the official
sought to be recalled.
• (a) Recall may be initiated by a preparatory recall assembly or by the registered voters
of the local government unit to which the local elective official subject to such recall • Section 71. Election on Recall. - Upon the filing of a valid resolution or petition for recall
belongs. with the appropriate local office of the COMELEC, the Commission or its duly authorized
representative shall set the date of the election on recall, which shall not be later than thirty
• (b) There shall be a preparatory recall assembly in every province, city, district, and
municipality which shall be composed of the following: (30) days after the filing of the resolution or petition for recall in the case of the barangay,
city, or municipal officials. and forty-five (45) days in the case of provincial officials. The
• (1) Provincial level. - All mayors, vice-mayors, and sanggunian members of the official or officials sought to be recalled shall automatically be considered as duly
municipalities and component cities;
Page 40 of 48
registered candidate or candidates to the pertinent positions and, like other candidates, 2 . ONG VS. ALEGRE- HE WAS DISQUALIFIED BUT HE SERVED FULL TERM
shall be entitled to be voted upon. THE TERM HE WAS DISQUALIFIED
• Section 72. Effectivity of Recall. - The recall of an elective local official shall be effective 3 . RIVERA VS. COMELEC (SAME AS ONG)
only upon the election and proclamation of a successor in the person of the candidate
receiving the highest number of votes cast during the election on recall. Should the official
sought to be recalled receive the highest number of votes, confidence in him is thereby 1. Latasa v COMELEC
affirmed, and he shall continue in office. 2. Ong v Alegre
• Section 73. Prohibition from Resignation. - The elective local official sought to be recalled 3. Rivera III v COMELEC
shall not be allowed to resign while the recall process is in progress.
4. Dizon v COMELEC
• Section 74. Limitations on Recall. - 5. Montebon v COMELEC
• (a) Any elective local official may be the subject of a recall election only once during his 6. Bolos v COMELEC
term of office for loss of confidence.
a) Leaving of an office because of candidacy for another office is voluntary
• (b) No recall shall take place within one (1) year from the date of the official's renunciation.
assumption to office or one (1) year immediately preceding a regular local election.
• Section 75. Expenses Incident to Recall Elections. - All expenses incident to recall
elections shall be borne by the COMELEC. For this purpose, there shall be included in the 7. Aldovino v COMELEC
annual General Appropriations Act a contingency fund at the disposal of the COMELEC for a) It is not voluntary renunciation when the candidate assumed office by
the conduct of recall elections. operation of law. (councilor to vice-mayor because the original vice-mayor
retired)
M. Term Limits
NO VIOLATION OF 3TLR 8. COMELEC v Cruz
1. SOCRATES VS. COMELEC- WHAT IS PROHIBITED IS THE IMMEDIATE a) 3 Term limit rule applies to barangay officials because it has been the intent of
ELECTION FOR 4TH TERM. HAGEDORN WAS 15 MONTHS OUT OF the Constitution.
OFFICE.
2. ADORMEO VS. COMELEC- 1992 (WIN) 1995(WIN) 1998(LOSE-RECALL)
3. MENDOZA VS. COMELEC (APPOINTED OIC GOV), ELECTED, RECALL, • SECTION 8. The term of office of elective local officials, except barangay officials, which
ELECTED, ELECTED shall be determined by law, shall be three years and no such official shall serve for more
4. DIZON VS. COMELEC- IS 2007 HIS 5TH TERM? HE WAS DISQUALIFIED than three consecutive terms. Voluntary renunciation of the office for any length of time
ON 3RD TERM shall not be considered as an interruption in the continuity of his service for the full term for
which he was elected.

VIOLATION
N. Katarungang Pambarangay
1 . LA TASA VS. COMELEC- HOLD OVER CAPACITY (MUNICIPALITY TO CITY)

Page 41 of 48
1. Pang-et v Manacnes-Dao-as • (c) A notice to constitute the lupon, which shall include the names of proposed members
a) Successful settlement is not a requisite in katarungang pambaranggay. Trying who have expressed their willingness to serve, shall be prepared by the punong
to settle is sufficient to satisfy the condition precedent. barangay within the first fifteen (15) days from the start of his term of office. Such notice
shall be posted in three (3) conspicuous places in the barangay continuously for a
period of not less than three (3) weeks;
2. Magno v Velasco v Jacoba • (d) The punong barangay, taking into consideration any opposition to the proposed
a) The rationale behind the personal appearance requirement is to enable appointment or any recommendations for appointment as may have been made within
the lupon to secure first hand and direct information about the facts and the period of posting, shall within ten (10) days thereafter, appoint as members those
issues,[8] the exception being in cases where minors or incompetents are whom he determines to be suitable therefor. Appointments shall be in writing, signed by
parties. There can be no quibbling that laymen of goodwill can easily agree to the punong barangay, and attested to by the barangay secretary.
conciliate and  settle their disputes between themselves without what • (e) The list of appointed members shall be posted in three (3) conspicuous places in the
sometimes is the unsettling assistance of lawyers whose presence could barangay for the entire duration of their term of office; and
sometimes obfuscate and confuse issues.[9]  Worse still, the participation of
lawyers with their penchant to use their analytical skills and legal knowledge • (f) In barangays where majority of the inhabitants are members of indigenous cultural
tend to prolong instead of expedite settlement of the case. communities, local systems of settling disputes through their councils of datus or elders
shall be recognized without prejudice to the applicable provisions of this Code.
b)         The prohibition against the presence of a lawyer in a barangay
• Section 400. Oath and Term of Office. - Upon appointment, each lupon member shall take
conciliation proceedings was not, to be sure, lost on respondent. Her defense an oath of office before the punong barangay. He shall hold office until a new lupon is
that the aforequoted Section 415 of the LGC does not apply since complainant constituted on the third year following his appointment unless sooner terminated by
addressed her Sumbong to the barangay captain of Brgy. San Pascual who resignation, transfer of residence or place of work, or withdrawal of appointment by the
thereafter proceeded to hear the same is specious at best. In this regard, punong barangay with the concurrence of the majority of all the members of the lupon.
suffice it to state that complainant wrote her Sumbong with the end in view of
availing herself of the benefits of barangay justice. That she addressed • Section 401. Vacancies. - Should a vacancy occur in the lupon for any cause, the punong
her Sumbong to the barangay captain is really of little moment since the latter barangay shall immediately appoint a qualified person who shall hold office only for the
chairs the Lupong Tagapamayapa. unexpired portion of the term.
• Section 402. Functions of the Lupon. - The lupon shall:
• (a) Exercise administrative supervision over the conciliation panels provided herein;
• Section 399. Lupong Tagapamayapa. - • (b) Meet regularly once a month to provide a forum for exchange of ideas among its
members and the public on matters relevant to the amicable settlement of disputes, and
• (a) There is hereby created in each barangay a lupong tagapamayapa, hereinafter to enable various conciliation panel members to share with one another their
referred to as the lupon, composed of the punong barangay, as chairman and ten (10) to observations and experiences in effecting speedy resolution of disputes; and
twenty (20) members. The lupon shall be constituted every three (3) years in the
manner provided herein. • (c) Exercise such other powers and perform such other duties and functions as may be
prescribed by law or ordinance.
• (b) Any person actually residing or working, in the barangay, not otherwise expressly
disqualified by law, and possessing integrity, impartiality, independence of mind, sense • Section 403. Secretary of the Lupon. - The barangay secretary shall concurrently serve as
of fairness, and reputation for probity, may be appointed a member of the lupon. the secretary of the lupon. He shall record the results of mediation proceedings before the
punong barangay and shall submit a report thereon to the proper city or municipal courts.

Page 42 of 48
He shall also receive and keep the records of proceedings submitted to him by the various involving questions of law to the punong barangay or any lupon or pangkat member
conciliation panels. whenever necessary in the exercise of his functions in the administration of the
katarungang pambarangay.
• Section 404. Pangkat ng Tagapagkasundo. -
• (a) There shall be constituted for each dispute brought before the lupon a conciliation • Section 408. Subject Matter for Amicable Settlement; Exception Thereto. - The lupon of
panel to be known as the pangkat ng tagapagkasundo, hereinafter referred to as the each barangay shall have authority to bring together the parties actually residing in the
pangkat, consisting of three (3) members who shall be chosen by the parties to the same city or municipality for amicable settlement of all disputes except:
dispute from the list of members of the lupon. • (a) Where one party is the government, or any subdivision or instrumentality thereof;
• Should the parties fail to agree on the pangkat membership, the same shall be • (b) Where one party is a public officer or employee, and the dispute relates to the
determined by lots drawn by the lupon chairman. performance of his official functions;
• (b) The three (3) members constituting the pangkat shall elect from among themselves • (c) Offenses punishable by imprisonment exceeding one (1) year or a fine exceeding
the chairman and the secretary. The secretary shall prepare the minutes of the pangkat Five thousand pesos (P5,000.00);
proceedings and submit a copy duly attested to by the chairman to the lupon secretary
• (d) Offenses where there is no private offended party;
and to the proper city or municipal court. He shall issue and cause to be served notices
to the parties concerned. • (e) Where the dispute involves real properties located in different cities or municipalities
unless the parties thereto agree to submit their differences to amicable settlement by an
• The lupon secretary shall issue certified true copies of any public record in his custody appropriate lupon;
that is not by law otherwise declared confidential.
• (f) Disputes involving parties who actually reside in barangays of different cities or
• Section 405. Vacancies in the Pangkat. - Any vacancy in the pangkat shall be chosen by municipalities, except where such barangay units adjoin each other and the parties
the parties to the dispute from among the other lupon members. Should the parties fail to thereto agree to submit their differences to amicable settlement by an appropriate lupon;
agree on a common choice, the vacancy shall be filled by lot to be drawn by the lupon
chairman. • (g) Such other classes of disputes which the President may determine in the interest of
Justice or upon the recommendation of the Secretary of Justice.
• Section 406. Character of Office and Service of Lupon Members. -
• The court in which non-criminal cases not falling within the authority of the lupon under
• (a) The lupon members, while in the performance of their official duties or on the this Code are filed may, at any time before trial motu propio refer the case to the lupon
occasion thereof, shall be deemed as persons in authority, as defined in the Revised concerned for amicable settlement.
Penal Code.
• Section 409. Venue. -
• (b) The lupon or pangkat members shall serve without compensation, except as
provided for in Section 393 and without prejudice to incentives as provided for in this • (a) Disputes between persons actually residing in the same barangay shall be brought
Section and in Book IV of this Code. The Department of the Interior and Local for amicable settlement before the lupon of said barangay.
Government shall provide for a system of granting economic or other incentives to the • (b) Those involving actual residents of different barangays within the same city or
lupon or pangkat members who adequately demonstrate the ability to judiciously and municipality shall be brought in the barangay where the respondent or any of the
expeditiously resolve cases referred to them. While in the performance of their duties, respondents actually resides, at the election of the complaint.
the lupon or pangkat members, whether in public or private employment, shall be
deemed to be on official time, and shall not suffer from any diminution in compensation • (c) All disputes involving real property or any interest therein shall be brought in the
barangay where the real property or the larger portion thereof is situated.
or allowance from said employment by reason thereof.
• Section 407. Legal Advice on Matters Involving Questions of Law. - The provincial, city
legal officer or prosecutor or the municipal legal officer shall render legal advice on matters
Page 43 of 48
• (d) Those arising at the workplace where the contending parties are employed or at the • (e) Period to arrive at a settlement - The pangkat shall arrive at a settlement or
institution where such parties are enrolled for study, shall be brought in the barangay resolution of the dispute within fifteen (15) days from the day it convenes in accordance
where such workplace or institution is located. with this section. This period shall, at the discretion of the pangkat, be extendible for
another period which shall not exceed fifteen (15) days, except in clearly meritorious
• Objections to venue shall be raised in the mediation proceedings before the punong
barangay; otherwise, the same shall be deemed waived. Any legal question which may cases.
confront the punong barangay in resolving objections to venue herein referred to may be • Section 411. Form of settlement. - All amicable settlements shall be in writing, in a
submitted to the Secretary of Justice, or his duly designated representative, whose language or dialect known to the parties, signed by them, and attested to by the lupon
ruling thereon shall be binding. chairman or the pangkat chairman, as the case may be. When the parties to the dispute
do not use the same language or dialect, the settlement shall be written in the language
• Section 410. Procedure for Amicable Settlement. -
known to them.
• (a) Who may initiate proceeding - Upon payment of the appropriate filing fee, any
individual who has a cause of action against another individual involving any matter • Section 412. Conciliation. -
within the authority of the lupon may complain, orally or in writing, to the lupon chairman • (a) Pre-condition to Filing of Complaint in Court. - No complaint, petition, action, or
of the barangay. proceeding involving any matter within the authority of the lupon shall be filed or
instituted directly in court or any other government office for adjudication, unless there
• (b) Mediation by lupon chairman - Upon receipt of the complaint, the lupon chairman
shall within the next working day summon the respondent(s), with notice to the has been a confrontation between the parties before the lupon chairman or the pangkat,
complainant(s) for them and their witnesses to appear before him for a mediation of their and that no conciliation or settlement has been reached as certified by the lupon
conflicting interests. If he fails in his mediation effort within fifteen (15) days from the first secretary or pangkat secretary as attested to by the lupon or pangkat chairman or
meeting of the parties before him, he shall forthwith set a date for the constitution of the unless the settlement has been repudiated by the parties thereto.
pangkat in accordance with the provisions of this Chapter. • (b) Where Parties May Go Directly to Court. - The parties may go directly to court in the
following instances:
• (c) Suspension of prescriptive period of offenses - While the dispute is under mediation,
conciliation, or arbitration, the prescriptive periods for offenses and cause of action • (1) Where the accused is under detention;
under existing laws shall be interrupted upon filing the complaint with the punong
• (2) Where a person has otherwise been deprived of personal liberty calling for habeas
barangay. The prescriptive periods shall resume upon receipt by the complainant of the corpus proceedings;
complainant or the certificate of repudiation or of the certification to file action issued by
the lupon or pangkat secretary: Provided, however, That such interruption shall not • (3) Where actions are coupled with provisional remedies such as preliminary
exceed sixty (60) days from the filing of the complaint with the punong barangay. injunction, attachment, delivery of personal property and support pendente lite; and
• (d) Issuance of summons; hearing; grounds for disqualification - The pangkat shall • (4) Where the action may otherwise be barred by the statute of limitations.
convene not later than three (3) days from its constitution, on the day and hour set by • (c) Conciliation among members of indigenous cultural communities. - The customs and
the lupon chairman, to hear both parties and their witnesses, simplify issues, and traditions of indigenous cultural communities shall be applied in settling disputes
explore all possibilities for amicable settlement. For this purpose, the pangkat may issue between members of the cultural communities.
summons for the personal appearance of parties and witnesses before it. In the event
that a party moves to disqualify any member of the pangkat by reason of relationship, • Section 413. Arbitration. -
bias, interest, or any other similar grounds discovered after the constitution of the • (a) The parties may, at any stage of the proceedings, agree in writing that they shall
pangkat, the matter shall be resolved by the affirmative vote of the majority of the abide by the arbitration award of the lupon chairman or the pangkat. Such agreement to
pangkat whose decision shall be final. Should disqualification be decided upon, the arbitrate may be repudiated within five (5) days from the date thereof for the same
resulting vacancy shall be filled as herein provided for. grounds and in accordance with the procedure hereinafter prescribed. The arbitration

Page 44 of 48
award shall be made after the lapse of the period for repudiation and within ten (10) • Section 420. Power to Administer Oaths. - The punong barangay, as chairman of the
days thereafter. lupong tagapamayapa, and the members of the pangkat are hereby authorized to
administer oaths in connection with any matter relating to all proceedings in the
• (b) The arbitration award shall be in writing in a language or dialect known to the parties.
When the parties to the dispute do not use the same language or dialect, the award implementation of the katarungang pambarangay.
shall be written in the language or dialect known to them. • Section 421. Administration; Rules and Regulations. - The city or municipal mayor, as the
case may be, shall see to the efficient and effective implementation and administration of
• Section 414. Proceedings Open to the Public; Exception. - All proceedings for settlement
shall be public and informal: Provided, however, That the lupon chairman or the pangkat the katarungang pambarangay. The Secretary of Justice shall promulgate the rules and
chairman, as the case may be, may motu proprio or upon request of a party, exclude the regulations necessary to implement this Chapter.
public from the proceedings in the interest of privacy, decency, or public morals. • Section 422. Appropriations. - Such amount as may be necessary for the effective
implementation of the katarungang pambarangay shall be provided for in the annual
• Section 415. Appearance of Parties in Person. - In all katarungang pambarangay
proceedings, the parties must appear in person without the assistance of counsel or budget of the city or municipality concerned.
representative, except for minors and incompetents who may be assisted by their next-of-
kin who are not lawyers. IV. League of Local Government Units
• Section 416. Effect of Amicable Settlement and Arbitration Award. - The amicable A. Liga ng mga Barangay
settlement and arbitration award shall have the force and effect of a final judgment of a
court upon the expiration of ten (10) days from the date thereof, unless repudiation of the 1. National Liga ng mga Barangay v Paredes
settlement has been made or a petition to nullify the award has been filed before the a) President has power of supervision over the leagues. However, the DILG
proper city or municipal court.
 cannot appoint a “caretaker” over the liga because it amounts to control.
However, this provision shall not apply to court cases settled by the lupon under the last
B. League of Municipalities
paragraph of Section 408 of this Code, in which case the compromise or the pangkat
chairman shall be submitted to the court and upon approval thereof, have the force and C. League of Cities
effect of a judgment of said court. D. League of Provinces
• Section 417. Execution. - The amicable settlement or arbitration award may be enforced
by execution by the lupon within six (6) months from the date of the settlement. After the
lapse of such time, the settlement may be enforced by action in the appropriate city or • Section 491. Purpose of Organization. - There shall be an organization of all barangays to
municipal court. be known as the liga ng mga barangay for the primary purpose of determining the
representation of the Liga in the sanggunians, and for ventilating, articulating and
• Section 418. Repudiation. - Any party to the dispute may, within ten (10) days from the crystallizing issues affecting barangay government administration and securing, through
date of the settlement, repudiate the same by filing with the lupon chairman a statement to proper and legal means, solutions thereto.
that effect sworn to before him, where the consent is vitiated by fraud, violence, or
intimidation. Such repudiation shall be sufficient basis for the issuance of the certification • Section 492. Representation, Chapters, National Liga. - Every barangay shall be
for filing a complaint as hereinabove provided. represented in said liga by the punong barangay, or in his absence or incapacity, by a
sanggunian member duly elected for the purpose among its members, who shall attend all
• Section 419. Transmittal of Settlement and Arbitration. - Award to the Court. - The meetings or deliberations called by the different chapters of the liga.

secretary of the lupon shall transmit the settlement or the arbitration award to the The liga shall have chapters at the municipal, city, provincial and metropolitan political
appropriate city or municipal court within five (5) days from the date of the award or from subdivision levels.

the lapse of the ten-day period repudiating the settlement and shall furnish copies thereof The municipal and city chapters of the liga shall be composed of the barangay
to each of the parties to the settlement and the lupon chairman.
Page 45 of 48
representatives of municipal and city barangays respectively. The duly elected presidents and crystallizing issues affecting municipal government administration, and securing,
of component municipal and city chapters shall constitute the provincial chapter or the through proper and legal means, solutions thereto.
metropolitan political subdivision chapter. The duly elected presidents of highly-urbanized
• The league shall form provincial chapters composed of the league presidents for all
cities, provincial chapters, the Metropolitan Manila chapter and metropolitan political component municipalities of the province.
subdivision chapters shall constitute the National Liga ng mga Barangay.
• Section 497. Representation. - Every municipality shall be represented in the league by
• Section 493. Organization. - The liga at the municipal, city, provincial, metropolitan the municipal mayor of in his absence, by the vice-mayor or a sanggunian member duly
political subdivision, and national levels directly elect a president, a vice-president, and five elected for the purpose by the members, who shall attend all meetings and participate in
(5) members of the board of directors. The board shall appoint its secretary and treasurer the deliberations of the league.
and create such other positions as it may deem necessary for the management of the
chapter. A secretary-general shall be elected from among the members of the national liga • Section 498. Powers, Functions and Duties of the League of Municipalities. - The league
and shall be charged with the overall operation of the liga on national level. The board of municipalities shall:
shall coordinate the activities of the chapters of the liga. • (a) Assist the national government in the formulation and implementation of the policies,
• Section 494. Ex-Officio Membership in Sanggunians. - The duly elected presidents of the programs and projects affecting municipalities as a whole;
liga at the municipal, city and provincial levels, including the component cities and • (b) Promote local autonomy at the municipal level;
municipalities of Metropolitan Manila, shall serve as ex-officio members of the
sangguniang bayan, sangguniang panlungsod, sangguniang panlalawigan, respectively. • (c) Adopt measures for the promotion of the welfare of all municipalities and its officials
and employees;
They shall serve as such only during their term of office as presidents of the liga chapters,
which in no case shall be beyond the term of office of the sanggunian concerned. • (d) Encourage people's participation in local government administration in order to
promote united and concerted action for the attainment of country-wide development
• Section 495. Powers, Functions and Duties of the Liga. - The liga shall: goals;
• (a) Give priority to programs designed for the total development of the barangays and in
consonance with the policies, programs and projects of the national government; • (e) Supplement the efforts of the national government in creating opportunities for
gainful employment within the municipalities;
• (b) Assist in the education of barangay residents for people's participation in local
government administration in order to promote united and concerted action to achieve • (f) Give priority to programs designed for the total development of the municipalities in
consonance with the policies, programs and projects of the national government;
country-wide development goals;
• (c) Supplement the efforts of government in creating gainful employment within the • (g) Serve as a forum for crystallizing and expressing ideas, seeking the necessary
assistance of the national government, and providing the private sector avenues for
barangay;
cooperation in the promotion of the welfare of the municipalities; and
• (d) Adopt measures to promote the welfare of barangay officials;
• (h) Exercise such other powers and perform such other duties and functions as the
• (e) Serve as a forum of the barangays in order to forge linkages with government and league may prescribe for the welfare of the municipalities.
non-governmental organizations and thereby promote the social, economic and political
well-being of the barangays; and • Section 499. Purpose of Organization. - There shall be an organization of all cities to be
known as the League of Cities for the primary purpose of ventilating, articulating and
• (f) Exercise such other powers and perform such other duties and functions which will crystallizing issues affecting city government administration, and securing, through proper
bring about stronger ties between barangays and promote the welfare of the barangay and legal means, solutions thereto.

inhabitants. The league may form chapters at the provincial level for the component cities of a
• Section 496. Purpose of Organization. - There shall be an organization of all municipalities province. Highly-urbanized cities may also form a chapter of the League. The National
to be known as league of municipalities for the primary purpose of ventilating, articulating
Page 46 of 48
League shall be composed of the presidents of the league of highly-urbanized cities and member duly elected for the purpose by the members, who shall attend all meetings and
the presidents of the provincial chapters of the league of component cities. participate in the deliberations of the league.
• Section 500. Representation. - Every city shall be represented in the league by the city • Section 504. Powers, Functions and Duties of the League of Provinces. - The league of
mayor or in his absence, by the city vice-mayor or a sanggunian member duly elected for provinces shall:
the purpose by the members, who shall attend all meetings and participate in the
• (a) Assist the national government in the formulation and implementation of the policies,
deliberations of the league. programs and projects affecting provinces as a whole;
• Section 501. Powers, Functions and Duties of the League of City. - The league of cities • (b) Promote local autonomy at the provincial level;
shall:
• (c) Adopt measures for the promotion of the welfare of all provinces and its officials and
• (a) Assist the national government in the formulation and implementation of the policies, employees;
programs and projects affecting cities as a whole;
• (d) Encourage people's participation in local government administration in order to
• (b) Promote local autonomy at the city level; promote united and concerted action for the attainment of countrywide employment
• (c) Adopt measures for the promotion of the welfare of all cities and its officials and within the province;
employees;
• (e) Supplement the efforts of the national government in creating opportunities for
• (d) Encourage people's participation in local government administration in order to gainful employment within the province;
promote united and concerted action for the attainment of country-wide development
• (f) Give priority to programs designed for the total development of the provinces in
goals; consonance with the policies, programs and projects of the national government;
• (e) Supplement the efforts of the national government in creating opportunities for • (g) Serve as a forum for crystallizing and expressing ideas, seeking the necessary
gainful employment the cities; assistance of the national government and providing the private sector avenues for
• (f) Give priority to programs designed for the total development of cities in consonance cooperation in the promotion of the welfare of the provinces; and
with the policies, programs and projects of the national government;
• (h) Exercise such other powers and perform such other duties and functions as the
• (g) Serve as a forum for crystallizing and expressing ideas, seeking the necessary league may prescribe for the welfare of the provinces and metropolitan political
assistance of the national government and providing the private sector avenues for subdivisions.
cooperation in the promotion of the welfare of the cities; and
• (h) Exercise such other powers and perform such other duties and functions as the • Provisions Common to All Leagues
league may prescribe for the welfare of the cities.
• Section 505. Funding.
• Section 502. Purpose of Organization. - There shall be an organization of all provinces to
be known as the League of Provinces for the primary purpose of ventilating, articulating • (a) All leagues shall derive its funds from contributions of member local government
and crystallizing issues affecting provincial and metropolitan political subdivision units and from fund-raising projects and activities without the necessity of securing
government administration, and securing, through proper and legal means, solutions permits therefor: Provided, That the proceeds from said fund-raising projects and
thereto. For this purpose, the Metropolitan Manila Area and any metropolitan political activities shall be used primarily to fund the projects for which the said proceeds have
subdivision shall be considered as separate provincial units of the league. been raised, subject to the pertinent provision of this Code and the pertinent provisions
of the Omnibus Election Code.
• Section 503. Representation. - Every province shall be represented in the league by the
provincial governor or in his absence, by the provincial vice-governor or a sanggunian • (b) All funds of leagues shall be deposited as trust funds with its treasurer and shall be
disbursed in accordance with the board of director's resolutions, subject to pertinent
Page 47 of 48
accounting and auditing rules and regulations: Provided, That the treasurer shall be • Section 509. Constitution and By-laws. - The leagues or federations shall adopt a
bonded in an amount to be determined by the board of directors. The funds of a chapter Constitution and by-laws which shall govern their internal organization and operation:
shall be deposited as chapter funds and funds of the national league shall be deposited Provided, That said Constitution and by-laws shall always conform to the provision of the
as national funds. Constitution and existing laws.
• Section 506. Organizational Structure. - To ensure the effective and efficient • Section 510. Funding. - The leagues and federations may derive funds from contributions
administration, the leagues for municipalities, cities and provinces shall elect chapter-level of individual league or federation members or from fund-raising projects or activities. The
and national-level boards of directors and a set of officers headed by the president. A local government unit concerned may appropriate funds to support the leagues or
secretary-general shall be chosen from among the national league members to manage federation organized pursuant to this Section, subject to the availability of funds.
the day to day operation and activities of the national league. The board of directors on the
chapter or national level may create such other positions as may be deemed necessary for
the management of the chapters and of the national league. The national board of
directors of the leagues for municipalities, cities or provinces shall coordinate programs,
projects and activities of chapter and the national-level league.
• Section 507. Constitution and By-laws of the Liga and the Leagues. - All other matters not
herein otherwise provided for affecting the internal organization of the leagues of local
government units shall be governed by their respective constitution and by-laws which are
hereby made suppletory to the provision of this Chapter: Provided, That said Constitution
and By-laws shall always conform to the provisions of the Constitution and existing laws.

Leagues and Federation of Local Elective Officials


• Section 508. Organization.
• (a) Vice-governors, vice-mayors, sanggunian members of barangays, municipalities,
component cities, highly-urbanized cities and provinces, and other elective local officials
of local government units, including those of the Metropolitan Manila Area and any
metropolitan political subdivisions, may form their respective leagues or federation,
subject to applicable provisions of this Title and pertinent provisions of this Code;
• (b) Sanggunian members of component cities and municipalities shall form a provincial
federation and elect a board of directors and a set of officers headed by the president.
The duly elected president of the provincial federation of sanggunian members of
component cities and municipalities shall be an ex-officio member of the sangguniang
panlalawigan concerned and shall serve as such only during his term of office as
president of the provincial federation of sanggunian members of component cities and
municipalities, which in no case shall be beyond the term of office of the sanggunian
panlalawigan concerned.

Page 48 of 48

You might also like